CME4PAC

Online CME for Physician Assistants

Menu

Skip to content
  • Home
  • Membership Benefits
  • My Account
  • About Us
  • Log In
  • Contact Us
  • Ortho on the Go

endocrine exam

Time limit: 0

Quiz-summary

0 of 30 questions completed

Questions:

  1. 1
  2. 2
  3. 3
  4. 4
  5. 5
  6. 6
  7. 7
  8. 8
  9. 9
  10. 10
  11. 11
  12. 12
  13. 13
  14. 14
  15. 15
  16. 16
  17. 17
  18. 18
  19. 19
  20. 20
  21. 21
  22. 22
  23. 23
  24. 24
  25. 25
  26. 26
  27. 27
  28. 28
  29. 29
  30. 30

Information

Approved for 3 Category 1 Self-Assessment Credits

Learning objectives: At the conclusion of the Endocrine Self-Assessment exam users should be aware of clinical signs of Addison’s disease, Cushing’s disease, and adrenal insufficiency, recognize signs and symptoms of IDDM and NIDDM, recognize the different types of insulin and oral medications to treat diabetes, recognize signs and symptoms of Multiple Endocrine Neoplasia (MEN) I and II. Users will also be able to recognize and treat gestational diabetes, thyroid disease, know the work-up of a thyroid nodule, diagnosis and manage Hashimoto’s thyroiditis, metabolic syndrome, and Graves disease, be able to differentiate between De Quervain’s (subacute) thyroiditis from other types of thyroiditis, be able to diagnose a pituitary adenoma, be able to recognize signs and symptoms of hyperthyroidism and hypothyroidism, and know the treatment for achondroplasia.

You have already completed the quiz before. Hence you can not start it again.

Quiz is loading...

You must sign in or sign up to start the quiz.

You have to finish following quiz, to start this quiz:

Results

0 of 30 questions answered correctly

Your time:

Time has elapsed

You have reached 0 of 0 points, (0)

Average score
 
 
Your score
 
 

Categories

  1. Not categorized 0%
  • You are required to complete the post-exam evaluation to receive CME credits. Please complete the post-exam evaluation here. Your CME certificate will be emailed to you shortly. If you don’t receive your certificate today, and the certificate email from CME4PAC is not located in your spam folder, please email, Dagan Cloutier, at dcloutier@cme4pac.com.

  1. 1
  2. 2
  3. 3
  4. 4
  5. 5
  6. 6
  7. 7
  8. 8
  9. 9
  10. 10
  11. 11
  12. 12
  13. 13
  14. 14
  15. 15
  16. 16
  17. 17
  18. 18
  19. 19
  20. 20
  21. 21
  22. 22
  23. 23
  24. 24
  25. 25
  26. 26
  27. 27
  28. 28
  29. 29
  30. 30
  1. Answered
  2. Review
  1. Question 1 of 30
    1. Question

    A 40-year-old female presents complaining of fatigue and diminished appetite resulting in an unwanted weight loss over the past several weeks.  When hungry, she reports having a desire for salty foods.  She has also been experiencing nausea, vomiting, and diarrhea more recently.  PMH is significant for IDDM.  Positive orthostatic vital signs and a temp of 99.9°.  Exam reveals an ill-appearing woman with dry mucosal membranes and the skin findings in the attached image.  Which of the following would you expect to find when performing serology?

    Correct

    The correct answer is D. The details of this case are most specific to acute adrenal insufficiency (Addison’s disease). Diabetic ketoacidosis (DKA) would result in hyperglycemia, elevated beta hydroxybutyrate (ketones), and likely normo/hypo-kalemia. DKA would not be ongoing for several weeks as was described in this case and also does not cause hyperpigmentation (see image). While fever and hypotension would be concerning for sepsis likely resulting in a leukocytosis, again, this is unlikely given the duration of symptoms.

    Addison’s disease is a rare disorder resulting in inadequate production of the body’s natural steroid hormones cortisol and aldosterone in the cortex of the adrenal glands. The result in an elevation of adrenocorticotropic hormone (ACTH) and melanocyte-stimulating hormone (MSH) due to the loss of negative feedback inhibition regulated by cortisol and aldosterone. The etiology is an acute medical stressor in the setting of an underlying autoimmune disorder. Underlying disorders that place a patient at risk include tuberculosis, fungal infections, cancer metastasis to the adrenal glands, bleeding into the adrenal gland, amyloidosis, type I diabetes, thyroiditis, and celiac disease. It is typically more common in females from age 30-50.

    Primary symptoms include fatigue, anorexia/weight loss, vomiting/diarrhea (low aldosterone impairs the absorption of sodium, chloride and water from the GI tract), dizziness, generalized abdominal pains, and hyperpigmentation (due to elevated MSH) of the face, palmar creases, buccal mucosa, and vermilion border of the lips. Behavior and mood changes may also occur. Vomiting and diarrhea fluid losses result in hyponatremia in up to 80% of cases. Hyperkalemia, hypoglycemia, and normocytic anemia are also usually found.

    The diagnosis is usually confirmed using the ACTH stimulation test or the insulin-induced hypoglycemia test may be used to determine if the symptoms of Addison’s disease are due to problems with the pituitary gland. A CT scan of the abdomen should be ordered to assess for enlargement of the adrenal glands and to detect other signs of disease such as calcification of the adrenal glands. Treatment includes aggressive fluid administration and a corticosteroid such as hydrocortisone, prednisone, or dexamethasone until vitals have stabilized and symptoms have improved. Maintenance includes lifelong glucocorticoids with occasional stress dosing required for acute illnesses or preoperatively.

    References:
    1. UpToDate
    2. American Academy of Family Practice: https://www.aafp.org/afp/2014/0401/p563.html
    3. https://rarediseases.org/rare-diseases/addisons-disease/
    4. https://www.ncbi.nlm.nih.gov/books/NBK441994/
    5. https://www.ncbi.nlm.nih.gov/books/NBK500031/

    Incorrect

    The correct answer is D. The details of this case are most specific to acute adrenal insufficiency (Addison’s disease). Diabetic ketoacidosis (DKA) would result in hyperglycemia, elevated beta hydroxybutyrate (ketones), and likely normo/hypo-kalemia. DKA would not be ongoing for several weeks as was described in this case and also does not cause hyperpigmentation (see image). While fever and hypotension would be concerning for sepsis likely resulting in a leukocytosis, again, this is unlikely given the duration of symptoms.

    Addison’s disease is a rare disorder resulting in inadequate production of the body’s natural steroid hormones cortisol and aldosterone in the cortex of the adrenal glands. The result in an elevation of adrenocorticotropic hormone (ACTH) and melanocyte-stimulating hormone (MSH) due to the loss of negative feedback inhibition regulated by cortisol and aldosterone. The etiology is an acute medical stressor in the setting of an underlying autoimmune disorder. Underlying disorders that place a patient at risk include tuberculosis, fungal infections, cancer metastasis to the adrenal glands, bleeding into the adrenal gland, amyloidosis, type I diabetes, thyroiditis, and celiac disease. It is typically more common in females from age 30-50.

    Primary symptoms include fatigue, anorexia/weight loss, vomiting/diarrhea (low aldosterone impairs the absorption of sodium, chloride and water from the GI tract), dizziness, generalized abdominal pains, and hyperpigmentation (due to elevated MSH) of the face, palmar creases, buccal mucosa, and vermilion border of the lips. Behavior and mood changes may also occur. Vomiting and diarrhea fluid losses result in hyponatremia in up to 80% of cases. Hyperkalemia, hypoglycemia, and normocytic anemia are also usually found.

    The diagnosis is usually confirmed using the ACTH stimulation test or the insulin-induced hypoglycemia test may be used to determine if the symptoms of Addison’s disease are due to problems with the pituitary gland. A CT scan of the abdomen should be ordered to assess for enlargement of the adrenal glands and to detect other signs of disease such as calcification of the adrenal glands. Treatment includes aggressive fluid administration and a corticosteroid such as hydrocortisone, prednisone, or dexamethasone until vitals have stabilized and symptoms have improved. Maintenance includes lifelong glucocorticoids with occasional stress dosing required for acute illnesses or preoperatively.

    References:
    1. UpToDate
    2. American Academy of Family Practice: https://www.aafp.org/afp/2014/0401/p563.html
    3. https://rarediseases.org/rare-diseases/addisons-disease/
    4. https://www.ncbi.nlm.nih.gov/books/NBK441994/
    5. https://www.ncbi.nlm.nih.gov/books/NBK500031/

  2. Question 2 of 30
    2. Question

    A 35-year-old female presents for evaluation of weight gain very infrequent menstrual periods over the past few months.  On review of systems, the patient admits to having several urinary tract infections during this time as well.  On physical exam, in addition to the attached image, you also notice a prominent dorsal fat pad.  Based on the suspected diagnosis, which test would likely be used in the workup of this patient?

    Correct

    The correct answer is B. This is the preferred test for the work-up of Cushing syndrome.  ACTH is utilized in adrenal insufficiency.  Glucose tolerance is ordered in the initial workup of possible new onset diabetes.  An elevated parathyroid hormone is present in hyperparathyroidism.  A thyroid stimulating hormone is used when screening for thyroid disorders and will be low in hyperthyroidism and elevated in hypothyroidism.

    Cushing’s syndrome is the result prolonged exposure to excess glucocorticoids. The etiologies can be separated into two categories: 1) ACTH-dependent which includes Cushing’s disease (pituitary hypersecretion of ACTH), nonpituitary tumors, nonhypothalamic tumors, or administration of exogenous ACTH (not glucocorticoids). 2) ACTH-independent: Iatrogenic administration of excessive amounts of synthetic glucocorticoids, adrenocortical adenomas and carcinomas, primary pigmented nodular adrenocortical disease and bilateral macronodular adrenal hyperplasia.

    In addition to the classic “moon facies”, facial plethora (“dusky, cyanosed and covered with a fine growth of hair”) and “buffalo hump” described in the case, other symptoms include decreased libido, obesity, menstrual changes, hirsutism, hypertension, ecchymoses/striae, lethargy, and depression.  Metabolic effects include hypertension, glucose intolerance, and hyperlipidemia.

    The workup starts with measuring plasma corticotropin (ACTH) on 2 occasions to differentiate between ACTH dependent and independent etiologies. Independent is confirmed If the ACTH is <5 pg/mL. A CT of the adrenal glands is the next step followed by MRI if not diagnostic.  An ACTH >20 pg/mL is consistent with ACTH-dependent Cushing’s syndrome.  To differentiate between pituitary and ectopic etiologies, two of the following tests in addition to a pituitary MRI are ordered: dexamethasone suppression test, a late-night salivary cortisol or 24-hour urinary free cortisol (UFC) excretion test.

    References:

    1. UpToDate
    2. https://www.ncbi.nlm.nih.gov/pmc/articles/PMC4407747/
    3. https://www.ncbi.nlm.nih.gov/pmc/articles/PMC4596033/
    Incorrect

    The correct answer is B. This is the preferred test for the work-up of Cushing syndrome.  ACTH is utilized in adrenal insufficiency.  Glucose tolerance is ordered in the initial workup of possible new onset diabetes.  An elevated parathyroid hormone is present in hyperparathyroidism.  A thyroid stimulating hormone is used when screening for thyroid disorders and will be low in hyperthyroidism and elevated in hypothyroidism.

    Cushing’s syndrome is the result prolonged exposure to excess glucocorticoids. The etiologies can be separated into two categories: 1) ACTH-dependent which includes Cushing’s disease (pituitary hypersecretion of ACTH), nonpituitary tumors, nonhypothalamic tumors, or administration of exogenous ACTH (not glucocorticoids). 2) ACTH-independent: Iatrogenic administration of excessive amounts of synthetic glucocorticoids, adrenocortical adenomas and carcinomas, primary pigmented nodular adrenocortical disease and bilateral macronodular adrenal hyperplasia.

    In addition to the classic “moon facies”, facial plethora (“dusky, cyanosed and covered with a fine growth of hair”) and “buffalo hump” described in the case, other symptoms include decreased libido, obesity, menstrual changes, hirsutism, hypertension, ecchymoses/striae, lethargy, and depression.  Metabolic effects include hypertension, glucose intolerance, and hyperlipidemia.

    The workup starts with measuring plasma corticotropin (ACTH) on 2 occasions to differentiate between ACTH dependent and independent etiologies. Independent is confirmed If the ACTH is <5 pg/mL. A CT of the adrenal glands is the next step followed by MRI if not diagnostic.  An ACTH >20 pg/mL is consistent with ACTH-dependent Cushing’s syndrome.  To differentiate between pituitary and ectopic etiologies, two of the following tests in addition to a pituitary MRI are ordered: dexamethasone suppression test, a late-night salivary cortisol or 24-hour urinary free cortisol (UFC) excretion test.

    References:

    1. UpToDate
    2. https://www.ncbi.nlm.nih.gov/pmc/articles/PMC4407747/
    3. https://www.ncbi.nlm.nih.gov/pmc/articles/PMC4596033/
  3. Question 3 of 30
    3. Question

    Which of the following is an adverse event resulting from the abrupt cessation of oral steroids if taken for more than 7-10 days?

    Correct

    The correct answer is B. Steroid withdrawal adrenal insufficiency or crisis is thought to be caused by a reduction in the secretion and synthesis of ACTH by the pituitary leading to adrenal atrophy and impaired responsiveness to ACTH.  This usually occurs if steroids have been used in excess of 2-3 weeks and stopped abruptly or with an inadequate tapering period.  Symptoms of steroid withdrawal are that same as those seen in adrenal insufficiency.  A suggested regimen to avoid this is to taper down by 5-10 mg/day every 1-2 weeks if prednisone was used at 40mg/day or greater (or equivalent to another steroid).  For prednisone given at lower doses, a smaller reduction per day over a longer period of time should be used.

    The major effects of endogenous glucocorticoids include skin thinning, ecchymosis, truncal obesity, facial enlargement, cataracts, osteoporosis, osteonecrosis, emotional lability, mania, depression, psychosis, delirium, confusion, hyperglycemia, immunocompromise, and leukocytosis.  Many of these will improve once the steroid is stopped but some effects may be permanent.  Administering steroids and NSAIDS simultaneously should be avoided when possible due to an increased risk of gastritis, ulcer and GI bleeding.

    Children are susceptible to growth impairment and even more susceptible to cataract formation.  These effects can even occur with use of inhaled steroids.  Use of steroids during pregnancy may result in premature rupture of the membranes, slowed fetal growth, hypertension, and gestational diabetes.  Steroids are, however, used to promote fetal lung maturity.  Glucocorticoids are considered compatible with breastfeeding, but it is best practice to discard breast milk for the first 4hours after use with a peak concentration approximately 2 hours after ingestion.

    References:

    1. https://www.jpsmjournal.com/article/S0885-3924(06)00718-4/fulltext
    2. https://www.ncbi.nlm.nih.gov/pmc/articles/PMC5682381/
    3. UpToDate
    Incorrect

    The correct answer is B. Steroid withdrawal adrenal insufficiency or crisis is thought to be caused by a reduction in the secretion and synthesis of ACTH by the pituitary leading to adrenal atrophy and impaired responsiveness to ACTH.  This usually occurs if steroids have been used in excess of 2-3 weeks and stopped abruptly or with an inadequate tapering period.  Symptoms of steroid withdrawal are that same as those seen in adrenal insufficiency.  A suggested regimen to avoid this is to taper down by 5-10 mg/day every 1-2 weeks if prednisone was used at 40mg/day or greater (or equivalent to another steroid).  For prednisone given at lower doses, a smaller reduction per day over a longer period of time should be used.

    The major effects of endogenous glucocorticoids include skin thinning, ecchymosis, truncal obesity, facial enlargement, cataracts, osteoporosis, osteonecrosis, emotional lability, mania, depression, psychosis, delirium, confusion, hyperglycemia, immunocompromise, and leukocytosis.  Many of these will improve once the steroid is stopped but some effects may be permanent.  Administering steroids and NSAIDS simultaneously should be avoided when possible due to an increased risk of gastritis, ulcer and GI bleeding.

    Children are susceptible to growth impairment and even more susceptible to cataract formation.  These effects can even occur with use of inhaled steroids.  Use of steroids during pregnancy may result in premature rupture of the membranes, slowed fetal growth, hypertension, and gestational diabetes.  Steroids are, however, used to promote fetal lung maturity.  Glucocorticoids are considered compatible with breastfeeding, but it is best practice to discard breast milk for the first 4hours after use with a peak concentration approximately 2 hours after ingestion.

    References:

    1. https://www.jpsmjournal.com/article/S0885-3924(06)00718-4/fulltext
    2. https://www.ncbi.nlm.nih.gov/pmc/articles/PMC5682381/
    3. UpToDate
  4. Question 4 of 30
    4. Question

    Which of the following should prompt suspicion of new onset insulin dependent diabetes mellitus?

    Correct

    The correct answer is A. Enuresis (bed wetting), especially in a child who was previously making it through the night, is a common initial presenting symptom in type I DM. Daytime polyuria, polydipsia, and weight loss are also common presenting symptoms in the previously undiagnosed diabetic.  Neuropathy is a chronic effect of uncontrolled DM and unexplained weight gain is usually a finding in non-insulin dependent DM.

    The primary risk factor for developing IDDM is family history.  Nearly half of previously undiagnosed childhood IDDM are asymptomatic.  Other common symptoms include polyuria, polydipsia, nocturia, and recent weight loss. Adolescent females may present with vaginal discharge due to candidiasis.  Lethargy or altered mental status may indicate diabetic ketoacidosis which is the primary initial presentation in approximately 30% of previously undiagnosed individuals.

    Workup includes a hemoglobin A1C, fasting plasma glucose (FPG), or an oral glucose tolerance test (OGTT). The benefit to the A1C test is that it can be performed without fasting or arranging for another time to take another test. A result of > 6.5% is considered abnormal and should be confirmed with another test.  The OGTT is more sensitive than the FPG.  An FPG ≥126 mg/dL is considered diagnostic for making the diagnosis of diabetes. An FPG  between 100 mg/dL to 125 mg/dL is consistent with prediabetes. An OGTT result of ≥200 mg/dL two hours after the glucose load is diagnostic of diabetes. A result of ≥140 to 199 mg/dL is consistent with prediabetes. Insulin therapy is the mainstay of treatment for IDDM.

    References:

    1. UpToDate
    2. https://www.niddk.nih.gov/health-information/diabetes/overview/what-is-diabetes/type-1-diabetes
    3. https://www.aace.com/disease-and-conditions/diabetes/type-1-diabetes
    Incorrect

    The correct answer is A. Enuresis (bed wetting), especially in a child who was previously making it through the night, is a common initial presenting symptom in type I DM. Daytime polyuria, polydipsia, and weight loss are also common presenting symptoms in the previously undiagnosed diabetic.  Neuropathy is a chronic effect of uncontrolled DM and unexplained weight gain is usually a finding in non-insulin dependent DM.

    The primary risk factor for developing IDDM is family history.  Nearly half of previously undiagnosed childhood IDDM are asymptomatic.  Other common symptoms include polyuria, polydipsia, nocturia, and recent weight loss. Adolescent females may present with vaginal discharge due to candidiasis.  Lethargy or altered mental status may indicate diabetic ketoacidosis which is the primary initial presentation in approximately 30% of previously undiagnosed individuals.

    Workup includes a hemoglobin A1C, fasting plasma glucose (FPG), or an oral glucose tolerance test (OGTT). The benefit to the A1C test is that it can be performed without fasting or arranging for another time to take another test. A result of > 6.5% is considered abnormal and should be confirmed with another test.  The OGTT is more sensitive than the FPG.  An FPG ≥126 mg/dL is considered diagnostic for making the diagnosis of diabetes. An FPG  between 100 mg/dL to 125 mg/dL is consistent with prediabetes. An OGTT result of ≥200 mg/dL two hours after the glucose load is diagnostic of diabetes. A result of ≥140 to 199 mg/dL is consistent with prediabetes. Insulin therapy is the mainstay of treatment for IDDM.

    References:

    1. UpToDate
    2. https://www.niddk.nih.gov/health-information/diabetes/overview/what-is-diabetes/type-1-diabetes
    3. https://www.aace.com/disease-and-conditions/diabetes/type-1-diabetes
  5. Question 5 of 30
    5. Question

    Lispro is an example of which type of insulin?

    Correct

    The correct answer is C. Other names of rapid-acting insulins include aspart and glulisine.  Glargine and detemir are examples of long acting. Regular insulin is the primary short-acting insulin and neutral protamine hagedorn is the primary intermediate-acting insulin.

    Insulin is naturally made in the beta cells of the Islets of Langerhans inside the pancreas. These islets also make glucagon from alpha cells.  In patients with insulin dependent diabetes mellitus (IDDM), insufficient insulin is produced, thus preventing sugar obtained from foods from being used or stored. In these cases, exogenous insulin is necessary.

    Rapid-acting starts working within approximately 15 minutes, peaks in about 1-2 hours, and last roughly 2-4 hours. There is also an inhaled version of rapid-acting insulin.  This type of insulin is typically given 5-15 minutes before a meal with a variable dose based on the carbohydrate content of food and the blood glucose level.  It is also the typical insulin used in insulin pumps to provide basal insulin levels.

    Short-acting starts working about 30 minutes after injection, peaks 2-3 hours after injection, and lasts for roughly 3-6 hours.  This type is typically reserved for scenarios such as gastroparesis or in cases of cystic fibrosis-related diabetes who are getting continuous overnight feedings.

    Intermediate-acting starts working in approximately 2-4 hours, peaks in 4-12 hours, and is lasts for about 12-18 hours.  These types of insulin have larger been replaced by long-acting versions.

    Long-acting begins working in 1.5-2 hours and lasts for approximately 1 day.  They are typically given in the evening with the purpose of providing insulin availability during the night and dampening the counterregulatory hormone response in the early morning.

    Ultra-long-acting starts to work in about 6 hours, does not peak, and lasts for at least 36 hours.  It has similar uses to long acting but may be more beneficial to certain patients depending on their lifestyle.

    Determining the insulin regime starts with estimating the total daily insulin requirement which is typically 0.3 to 1 units/kg/day. Next, the basal insulin (insulin glargine, detemir, or degludec or rate of pump) dose needs to be calculated. In children, this is roughly 50% of the total daily dose but may be lower in children during growth spurts.  This is typically administered once in the evening if injecting or divided by 24 to determine the hourly rate if using a pump. In addition, a dose of rapid-acting insulin is given before each meal or snack based on the estimated carbohydrate intake.  Divide the patient’s total daily insulin dose by 500 to determine the carbohydrate ratio.  The total estimated carbohydrates to be ingested is then divided by this ratio number to determine how many units are needed.

     

    References:

    1. UpToDate
    2. https://www.diabetes.org/healthy-living/medication-treatments/insulin-other-injectables/insulin-basics
    3. https://fpnotebook.com/endo/pharm/InslnDsngInTypDbts.htm
    Incorrect

    The correct answer is C. Other names of rapid-acting insulins include aspart and glulisine.  Glargine and detemir are examples of long acting. Regular insulin is the primary short-acting insulin and neutral protamine hagedorn is the primary intermediate-acting insulin.

    Insulin is naturally made in the beta cells of the Islets of Langerhans inside the pancreas. These islets also make glucagon from alpha cells.  In patients with insulin dependent diabetes mellitus (IDDM), insufficient insulin is produced, thus preventing sugar obtained from foods from being used or stored. In these cases, exogenous insulin is necessary.

    Rapid-acting starts working within approximately 15 minutes, peaks in about 1-2 hours, and last roughly 2-4 hours. There is also an inhaled version of rapid-acting insulin.  This type of insulin is typically given 5-15 minutes before a meal with a variable dose based on the carbohydrate content of food and the blood glucose level.  It is also the typical insulin used in insulin pumps to provide basal insulin levels.

    Short-acting starts working about 30 minutes after injection, peaks 2-3 hours after injection, and lasts for roughly 3-6 hours.  This type is typically reserved for scenarios such as gastroparesis or in cases of cystic fibrosis-related diabetes who are getting continuous overnight feedings.

    Intermediate-acting starts working in approximately 2-4 hours, peaks in 4-12 hours, and is lasts for about 12-18 hours.  These types of insulin have larger been replaced by long-acting versions.

    Long-acting begins working in 1.5-2 hours and lasts for approximately 1 day.  They are typically given in the evening with the purpose of providing insulin availability during the night and dampening the counterregulatory hormone response in the early morning.

    Ultra-long-acting starts to work in about 6 hours, does not peak, and lasts for at least 36 hours.  It has similar uses to long acting but may be more beneficial to certain patients depending on their lifestyle.

    Determining the insulin regime starts with estimating the total daily insulin requirement which is typically 0.3 to 1 units/kg/day. Next, the basal insulin (insulin glargine, detemir, or degludec or rate of pump) dose needs to be calculated. In children, this is roughly 50% of the total daily dose but may be lower in children during growth spurts.  This is typically administered once in the evening if injecting or divided by 24 to determine the hourly rate if using a pump. In addition, a dose of rapid-acting insulin is given before each meal or snack based on the estimated carbohydrate intake.  Divide the patient’s total daily insulin dose by 500 to determine the carbohydrate ratio.  The total estimated carbohydrates to be ingested is then divided by this ratio number to determine how many units are needed.

     

    References:

    1. UpToDate
    2. https://www.diabetes.org/healthy-living/medication-treatments/insulin-other-injectables/insulin-basics
    3. https://fpnotebook.com/endo/pharm/InslnDsngInTypDbts.htm
  6. Question 6 of 30
    6. Question

    Which of the following is the most common presenting symptoms in patients with new onset non-insulin dependent diabetes mellitus (NIDDM)?

    Correct

    The correct answer is A. All of these are potential complaints that a patient may present with when initially being diagnosed with NIDDM. However, the vast majority are asymptomatic.

    NIDDM, or type 2 diabetes, accounts for approximately 90% of all cases of diabetes.  It is the result of insulin deficiency and/or resistance.  Most of these diagnoses are made when hyperglycemia is found on routine testing which triggers a more in-depth workup. Unlike insulin dependent (type 1 DM), type 2 rarely presents with, or results in, diabetic ketoacidosis (DKA) however, some can develop a hyperosmolar state, with significant hyperglycemia, severe dehydration, and altered mental status, but without ketoacidosis. In an asymptomatic individual, the diagnosis of diabetes can be made using one of the following 3 methods:

    • Fasting (no calories for ≥ 8 hours) blood glucose of ≥126 mg/dL
    • Two-hour glucose of ≥200 mg/dL during a 75 g oral glucose tolerance test (OGTT)
    • Hemoglobin A1C values ≥6.5 %

    In symptomatic patients, a random glucose ≥200 mg/dL can be used.

    The primary risk factors for the development of NIDDM are family history, race (black, Asian, Hispanic), and obesity.  Of the 3 primary risk factors, obesity has the greatest causative impact.  Rare primary causes include exposure to arsenic, polycarbonate plastics, epoxy resins, organophosphates, and chlorinated pesticides. Some medical conditions have been associated with secondarily developing diabetes.  These include a prior history of gestational diabetes, heart failure, myocardial infarction, hyperuricemia, metabolic syndrome, and polycystic ovarian syndrome. Risk modifiers include exercise, diet, smoking, sleep patterns, and breast feeding.

     

    References:

    1. UpToDate
    2. https://www.niddk.nih.gov/health-information/diabetes/overview/risk-factors-type-2-diabetes
    3. https://www.ncbi.nlm.nih.gov/pmc/articles/PMC7503727/
    Incorrect

    The correct answer is A. All of these are potential complaints that a patient may present with when initially being diagnosed with NIDDM. However, the vast majority are asymptomatic.

    NIDDM, or type 2 diabetes, accounts for approximately 90% of all cases of diabetes.  It is the result of insulin deficiency and/or resistance.  Most of these diagnoses are made when hyperglycemia is found on routine testing which triggers a more in-depth workup. Unlike insulin dependent (type 1 DM), type 2 rarely presents with, or results in, diabetic ketoacidosis (DKA) however, some can develop a hyperosmolar state, with significant hyperglycemia, severe dehydration, and altered mental status, but without ketoacidosis. In an asymptomatic individual, the diagnosis of diabetes can be made using one of the following 3 methods:

    • Fasting (no calories for ≥ 8 hours) blood glucose of ≥126 mg/dL
    • Two-hour glucose of ≥200 mg/dL during a 75 g oral glucose tolerance test (OGTT)
    • Hemoglobin A1C values ≥6.5 %

    In symptomatic patients, a random glucose ≥200 mg/dL can be used.

    The primary risk factors for the development of NIDDM are family history, race (black, Asian, Hispanic), and obesity.  Of the 3 primary risk factors, obesity has the greatest causative impact.  Rare primary causes include exposure to arsenic, polycarbonate plastics, epoxy resins, organophosphates, and chlorinated pesticides. Some medical conditions have been associated with secondarily developing diabetes.  These include a prior history of gestational diabetes, heart failure, myocardial infarction, hyperuricemia, metabolic syndrome, and polycystic ovarian syndrome. Risk modifiers include exercise, diet, smoking, sleep patterns, and breast feeding.

     

    References:

    1. UpToDate
    2. https://www.niddk.nih.gov/health-information/diabetes/overview/risk-factors-type-2-diabetes
    3. https://www.ncbi.nlm.nih.gov/pmc/articles/PMC7503727/
  7. Question 7 of 30
    7. Question

    A 45-year-old female with no known medical history presents with a complaint of urinary frequency.  Physical exam reveals an obese female in no acute distress with stable vitals.  A urinalysis reveals no WBC’s, RBC’s, leukocyte esterase or nitrates but the glucose was 220 mg/dL.  An oral glucose tolerance test (OGTT) confirms a new diagnosis of type 2 diabetes.  What is the appropriate initial treatment?

    Correct

    The correct answer is E. Type 2 (NIDDM) diabetes is usually initially treated with metformin (Glucophage) as opposed to insulin since it is usually the result of decreased insulin sensitivity as opposed to insufficient insulin from the pancreas.  Once the HbA1C level rises above 7.5% while on a medication, combination therapy with two oral agents, or by adding insulin to a single agent, should be considered.

    Metformin is a biguanide which causes hepatic uptake of glucose and inhibits gluconeogenesis via the mitocho-=[ndrial enzymes. It generally does not result in significant hypoglycemia or weight gain and has demonstrated an ability to delay the progression of NIDDM as well as sensitizing peripheral tissues to insulin and lower lipid levels. It is contraindicated in advanced renal insufficiency and may cause deficiencies of both vitamin B12 and folic acid.

    Liraglutide (Victoza) is a GLP-1 analog which targets the incretin system which generally results in a 1.5% decrease in A1C. Liraglutide has been shown to decrease body weight when given in combination with metformin and can help lower systolic blood pressure.

    Empagliflozin (Jardiance) is a sodium-glucose cotransporter (SGLT2) inhibitor which lowers blood sugar by blocking glucose reabsorption in the proximal renal tubule. It may be most effective for patients who have lost pancreatic β-cells.  Weight loss and blood pressure reduction may be an added benefit while urinary tract infections possibly advancing to urosepsis can occur.

    Glipizide (Glocotrol) is a 2nd generation sulfonylurea which causes increased insulin secretion and limits gluconeogenesis in the liver. In addition, it decreases the breakdown of lipids to fatty acids and reduces the clearance of insulin in the liver, thereby allowing for it to continue working systemically. It is generally a second-line option. Hypoglycemia is the major side effect and it’s use is contraindicated in pregnant patients and those with hepatic and renal disease and should be used with caution in patients on beta blockers.

     

    References:

    1. https://www.ncbi.nlm.nih.gov/books/NBK557441/
    2. https://www.ncbi.nlm.nih.gov/books/NBK245/
    3. https://www.uptodate.com/contents/initial-management-of-hyperglycemia-in-adults-with-type-2-diabetes-mellitus?search=type%202%20diabetes%20treatment&source=search_result&selectedTitle=1~150&usage_type=default&display_rank=1#H26275125
    4. https://www.frontiersin.org/articles/10.3389/fendo.2017.00006/full
    Incorrect

    The correct answer is E. Type 2 (NIDDM) diabetes is usually initially treated with metformin (Glucophage) as opposed to insulin since it is usually the result of decreased insulin sensitivity as opposed to insufficient insulin from the pancreas.  Once the HbA1C level rises above 7.5% while on a medication, combination therapy with two oral agents, or by adding insulin to a single agent, should be considered.

    Metformin is a biguanide which causes hepatic uptake of glucose and inhibits gluconeogenesis via the mitocho-=[ndrial enzymes. It generally does not result in significant hypoglycemia or weight gain and has demonstrated an ability to delay the progression of NIDDM as well as sensitizing peripheral tissues to insulin and lower lipid levels. It is contraindicated in advanced renal insufficiency and may cause deficiencies of both vitamin B12 and folic acid.

    Liraglutide (Victoza) is a GLP-1 analog which targets the incretin system which generally results in a 1.5% decrease in A1C. Liraglutide has been shown to decrease body weight when given in combination with metformin and can help lower systolic blood pressure.

    Empagliflozin (Jardiance) is a sodium-glucose cotransporter (SGLT2) inhibitor which lowers blood sugar by blocking glucose reabsorption in the proximal renal tubule. It may be most effective for patients who have lost pancreatic β-cells.  Weight loss and blood pressure reduction may be an added benefit while urinary tract infections possibly advancing to urosepsis can occur.

    Glipizide (Glocotrol) is a 2nd generation sulfonylurea which causes increased insulin secretion and limits gluconeogenesis in the liver. In addition, it decreases the breakdown of lipids to fatty acids and reduces the clearance of insulin in the liver, thereby allowing for it to continue working systemically. It is generally a second-line option. Hypoglycemia is the major side effect and it’s use is contraindicated in pregnant patients and those with hepatic and renal disease and should be used with caution in patients on beta blockers.

     

    References:

    1. https://www.ncbi.nlm.nih.gov/books/NBK557441/
    2. https://www.ncbi.nlm.nih.gov/books/NBK245/
    3. https://www.uptodate.com/contents/initial-management-of-hyperglycemia-in-adults-with-type-2-diabetes-mellitus?search=type%202%20diabetes%20treatment&source=search_result&selectedTitle=1~150&usage_type=default&display_rank=1#H26275125
    4. https://www.frontiersin.org/articles/10.3389/fendo.2017.00006/full
  8. Question 8 of 30
    8. Question

    A patient presents with fever and is very ill-appearing.  You suspect the patient has sepsis.  Which of the following basic metabolic components is most likely to be abnormal?

    Correct

    The correct answer is B. Sepsis is a common cause of hypoglycemia.  Other causes of hypoglycemia aside from diabetes medications include alcohol binge drinking, renal insufficiency, mechanical ventilation, severity of medical illness, malnourishment, and Addison’s disease.  Unexplained cases may also be the result of an insulinoma or malicious/accidental administration of insulin or diabetic pills.

    When working up a patient with hypoglycemia unrelated to known excess insulin or oral DM meds, a detailed history is the first step in determining the direction of the work-up.  A basic work-up for unexplained hypoglycemia includes a glucose, insulin antibodies, C-peptide, beta-hydroxybutyrate, proinsulin, sulfonylurea and meglitinide levels.  Other tests that can be utilized include a mixed-meal postprandial test and a 72-hour fasting evaluation.  CT, MRI, or ultrasound are helpful in the work-up for an insulinoma.

    Plasma insulin, C-peptide, and proinsulin values are elevated in patients with insulinoma, oral hypoglycemia agent-induced hypoglycemia, and insulin autoimmune hypoglycemia.  Oral hypoglycemic medications will result in elevated sulfonylurea or meglitinide levels. Insulin autoimmune hypoglycemia can be differentiated from insulinoma by the presence of insulin or insulin antibodies. Elevations of plasma insulin, C-peptide, and proinsulin are consistent with a noninsulinoma pancreatogenous hypoglycemia syndrome (NIPHS). This is caused by islet cell hypertrophy with neodifferentiation of islet of Langerhans cells from pancreatic duct epithelium. Typically, this is the cause of an event which occurs 2-4 hours after a meal.  In cases where plasma insulin is high, and C-peptide and proinsulin levels are low, exogenous insulin administration is likely.

    If able, a patient with hypoglycemia should consume 15-20 grams of fast-acting carbohydrates, which include glucose tablet, juice/soda, raisin, saltine crackers, table sugar, honey or hard candies. In know diabetics who are taking an alpha-glucosidase inhibitor, glucose tablets will be most effective because the other options contain disaccharides, and the alpha-glucosidase slows digestion of these.  If a patient is unable to eat, IV glucose or IM glucagon should be given.  An infusion may be required if blood sugar levels are unable to be kept within normal range.

    References:

    1. UpToDate
    2. https://www.ncbi.nlm.nih.gov/books/NBK355894/
    Incorrect

    The correct answer is B. Sepsis is a common cause of hypoglycemia.  Other causes of hypoglycemia aside from diabetes medications include alcohol binge drinking, renal insufficiency, mechanical ventilation, severity of medical illness, malnourishment, and Addison’s disease.  Unexplained cases may also be the result of an insulinoma or malicious/accidental administration of insulin or diabetic pills.

    When working up a patient with hypoglycemia unrelated to known excess insulin or oral DM meds, a detailed history is the first step in determining the direction of the work-up.  A basic work-up for unexplained hypoglycemia includes a glucose, insulin antibodies, C-peptide, beta-hydroxybutyrate, proinsulin, sulfonylurea and meglitinide levels.  Other tests that can be utilized include a mixed-meal postprandial test and a 72-hour fasting evaluation.  CT, MRI, or ultrasound are helpful in the work-up for an insulinoma.

    Plasma insulin, C-peptide, and proinsulin values are elevated in patients with insulinoma, oral hypoglycemia agent-induced hypoglycemia, and insulin autoimmune hypoglycemia.  Oral hypoglycemic medications will result in elevated sulfonylurea or meglitinide levels. Insulin autoimmune hypoglycemia can be differentiated from insulinoma by the presence of insulin or insulin antibodies. Elevations of plasma insulin, C-peptide, and proinsulin are consistent with a noninsulinoma pancreatogenous hypoglycemia syndrome (NIPHS). This is caused by islet cell hypertrophy with neodifferentiation of islet of Langerhans cells from pancreatic duct epithelium. Typically, this is the cause of an event which occurs 2-4 hours after a meal.  In cases where plasma insulin is high, and C-peptide and proinsulin levels are low, exogenous insulin administration is likely.

    If able, a patient with hypoglycemia should consume 15-20 grams of fast-acting carbohydrates, which include glucose tablet, juice/soda, raisin, saltine crackers, table sugar, honey or hard candies. In know diabetics who are taking an alpha-glucosidase inhibitor, glucose tablets will be most effective because the other options contain disaccharides, and the alpha-glucosidase slows digestion of these.  If a patient is unable to eat, IV glucose or IM glucagon should be given.  An infusion may be required if blood sugar levels are unable to be kept within normal range.

    References:

    1. UpToDate
    2. https://www.ncbi.nlm.nih.gov/books/NBK355894/
  9. Question 9 of 30
    9. Question

    A 45-year-old male reports a gradual progression of decreased energy level and a depressed affect over the past 1-2 years.  He does not have a primary care provider, so he has delayed seeking care for these issues.  Physical exam reveals normal vital signs but what appears to be less general muscle mass and body hair than expected.  Which of the following tests would be most beneficial in making a diagnosis?

    Correct

    The correct answer is C. The history and exam are most consistent with a diagnosis of hypogonadism.  A CBC would be useful to rule out anemia which would cause a diminished energy level but not the other findings.  A cortisol level would be indicated if Cushing’s Syndrome was suspected to cause depression, however, it usually results in weight gain and edema.  Vitamin D deficiency is commonly asymptomatic with bone pain being the most common symptom.

    Hypogonadism is the result of a decline in testicular function causing a decrease in sperm and/or testosterone production.  Primary hypogonadism refers to a disorder involving the testes whereas secondary is a result of a disorder involving the hypothalamus or pituitary.

    The underlying problem can develop during pregnancy (1st or 3rd trimester), prepubertal, or during adulthood.  If the problem occurs during gestational fetal development, ambiguous genitalia or a micropenis can occur at birth. If it occurs during the first decade of life, the child will not have a full or any puberty.  During adulthood, the result is a decline in energy and libido. If not treated, it can lead to a loss of pubic hair, muscle mass, bone mineral density hot flashes, gynecomastia and infertility.

    The work-up starts with a fasting total testosterone level drawn between 8-10 AM.  If low, the test should be repeated at least one additional time and if still low, serum luteinizing hormone (LH) and follicle-stimulating hormone (FSH) concentrations should be measured to distinguish primary from secondary hypogonadism. The primary options for treating hypogonadism include testosterone gel or intramuscular preparations.  The decision can be influenced by patient choice, cost, convenience, and insurance coverage.  Most cases of hypogonadism require lifelong treatment.  Polycythemia can be a complication of testosterone replacement, therefore, monitor hematocrit values annually. A prostate exam and prostate-specific antigen (PSA) measurements should be performed before testosterone therapy and periodically after treatment is instituted.

    References:

    1. UpToDate
    2. https://www.ncbi.nlm.nih.gov/pmc/articles/PMC3255409/
    3. https://emedicine.medscape.com/article/922038-overview
    Incorrect

    The correct answer is C. The history and exam are most consistent with a diagnosis of hypogonadism.  A CBC would be useful to rule out anemia which would cause a diminished energy level but not the other findings.  A cortisol level would be indicated if Cushing’s Syndrome was suspected to cause depression, however, it usually results in weight gain and edema.  Vitamin D deficiency is commonly asymptomatic with bone pain being the most common symptom.

    Hypogonadism is the result of a decline in testicular function causing a decrease in sperm and/or testosterone production.  Primary hypogonadism refers to a disorder involving the testes whereas secondary is a result of a disorder involving the hypothalamus or pituitary.

    The underlying problem can develop during pregnancy (1st or 3rd trimester), prepubertal, or during adulthood.  If the problem occurs during gestational fetal development, ambiguous genitalia or a micropenis can occur at birth. If it occurs during the first decade of life, the child will not have a full or any puberty.  During adulthood, the result is a decline in energy and libido. If not treated, it can lead to a loss of pubic hair, muscle mass, bone mineral density hot flashes, gynecomastia and infertility.

    The work-up starts with a fasting total testosterone level drawn between 8-10 AM.  If low, the test should be repeated at least one additional time and if still low, serum luteinizing hormone (LH) and follicle-stimulating hormone (FSH) concentrations should be measured to distinguish primary from secondary hypogonadism. The primary options for treating hypogonadism include testosterone gel or intramuscular preparations.  The decision can be influenced by patient choice, cost, convenience, and insurance coverage.  Most cases of hypogonadism require lifelong treatment.  Polycythemia can be a complication of testosterone replacement, therefore, monitor hematocrit values annually. A prostate exam and prostate-specific antigen (PSA) measurements should be performed before testosterone therapy and periodically after treatment is instituted.

    References:

    1. UpToDate
    2. https://www.ncbi.nlm.nih.gov/pmc/articles/PMC3255409/
    3. https://emedicine.medscape.com/article/922038-overview
  10. Question 10 of 30
    10. Question

    A 30-year-old man presents with a chief complaint of polyuria and polydipsia.  Review of systems is positive only for fairly regular episodes of constipation.  Past medical history positive for a single episode of ureterolithiasis.  Basic metabolic panel results: Sodium 146 (136–145); Potassium 3.4 (3.4–5.4); Chloride 104 (98–107); Calcium 14.7 (8.7–10.4); Bicarbonate 28 (23–29); BUN 30 (6–20); Creatinine 1.1 (0.5–1.1).  What is the most likely etiology of the patients’ symptoms?

    Correct

    The correct answer is C.  The symptoms and elevated calcium are consistent with a new diagnosis of MEN 1.  DM should be at the top of the differential diagnosis based on the complaint, but a normal blood sugar makes this unlikely.  MEN 2b does not affect the parathyroid making it unlikely with the hypercalcemia.  Wilson disease is a genetic disorder of copper metabolism which can occasionally cause hypocalcemia.

    Multiple Endocrine Neoplasia (MEN) is a hereditary disorder which results in various endocrine gland tumors.  The clinical and laboratory effects vary depending on which glands are most effected.  MEN type I is a genetic mutation of tumor suppressor MEN gene found on chromosome 11 and effects the parathyroid glad at the greatest rate with the pituitary, pancreas, and stomach being possible as well.  Given the small anatomic area, pituitary masses cause the earliest symptoms which can include headaches and vision changes and are usually detected due to these complaints prior to causing hypogonadism.  Hyperparathyroidism is the most common manifestation and occurs in the patients’ 20’s-40’s. Typically, it is the hypercalcemia found on routine lab work that is the initial finding in an otherwise asymptomatic patient.  When symptomatic, a patient may complain of kidney stones, polyuria, polydipsia, or constipation.  Insulinomas result in persistent hypoglycemia and gastrinomas cause multiple gastric ulcers known as Zollinger-Ellison syndrome.  MEN is usually a clinical diagnosis based upon the occurrence of two or more tumor in the anatomic areas mentioned above. Laboratory tests that may be ordered include fasting serum glucose/insulin, serum C peptide, calcium-stimulated gastrin, and serum proinsulin. Surgical resection is generally the most effective treatment.

     

    References:

    1. https://bestpractice.bmj.com/topics/en-us/866
    2. UpToDate
    Incorrect

    The correct answer is C.  The symptoms and elevated calcium are consistent with a new diagnosis of MEN 1.  DM should be at the top of the differential diagnosis based on the complaint, but a normal blood sugar makes this unlikely.  MEN 2b does not affect the parathyroid making it unlikely with the hypercalcemia.  Wilson disease is a genetic disorder of copper metabolism which can occasionally cause hypocalcemia.

    Multiple Endocrine Neoplasia (MEN) is a hereditary disorder which results in various endocrine gland tumors.  The clinical and laboratory effects vary depending on which glands are most effected.  MEN type I is a genetic mutation of tumor suppressor MEN gene found on chromosome 11 and effects the parathyroid glad at the greatest rate with the pituitary, pancreas, and stomach being possible as well.  Given the small anatomic area, pituitary masses cause the earliest symptoms which can include headaches and vision changes and are usually detected due to these complaints prior to causing hypogonadism.  Hyperparathyroidism is the most common manifestation and occurs in the patients’ 20’s-40’s. Typically, it is the hypercalcemia found on routine lab work that is the initial finding in an otherwise asymptomatic patient.  When symptomatic, a patient may complain of kidney stones, polyuria, polydipsia, or constipation.  Insulinomas result in persistent hypoglycemia and gastrinomas cause multiple gastric ulcers known as Zollinger-Ellison syndrome.  MEN is usually a clinical diagnosis based upon the occurrence of two or more tumor in the anatomic areas mentioned above. Laboratory tests that may be ordered include fasting serum glucose/insulin, serum C peptide, calcium-stimulated gastrin, and serum proinsulin. Surgical resection is generally the most effective treatment.

     

    References:

    1. https://bestpractice.bmj.com/topics/en-us/866
    2. UpToDate
  11. Question 11 of 30
    11. Question

    A 50-year-old male presents with intermittent episodes of feelings of anxiety, including palpitations and diaphoresis. He has borrowed his wife’s home blood pressure monitor on several occasions and has noticed his readings have been remarkably high. Which underlying disorder is most likely responsible for the diagnosis?

    Correct

    The correct answer is D. The patients’ symptoms are most consistent with a pheochromocytoma and up to fifty percent of pheochromocytoma’s are due to multiple endocrine neoplasia type 2 (MEN II). Aortic dissection generally does not cause palpitations and might result in a noticeable disparity when comparing BP’s of the upper extremities. A hyperthyroid storm might present similarly however hypothyroidism would not. An insulinoma would be found in MEN type 1 and would result in episodes of hypoglycemia.  Panic attacks are generally a diagnosis of exclusion and should not result in severe blood pressure elevations.

    Multiple Endocrine Neoplasia (MEN) is a hereditary disorder which results in various endocrine gland tumors.  The clinical and laboratory effects vary depending on which glands are most effected. There are 2 types of MEN with the second being divided into 2 subtypes.  MEN type 2 is a genetic mutation of RET gene which causes a protooncogene to convert to an oncogene. Type 2a produces tumors of the medullary thyroid, adrenal gland (pheochromocytoma as described in this case), and parathyroid.  Type IIb results in multiple neuromas (masses of tongue, lips, roof of mouth) instead of parathyroid involvement. MEN Type I involves the parathyroid, pituitary, & pancreas.

    Workup considerations include serum calcitonin, carcinoembryonic antigen, urine and serum metanephrines, parathyroid hormone and calcium.  Metanephrine testing is most specific to the workup of pheochromocytoma which was the most likely cause of the patients’ symptoms in this question. CT, MRI, and/or ultrasound are beneficial in determining the specific location and complexity of a tumor.  Surgical resection is generally recommended in all patients with MEN. First and second-degree family members should be screened for MEN once the diagnosis has been established in a relative.

    References:

    1. https://bestpractice.bmj.com/topics/en-us/866
    2. UpToDate
    Incorrect

    The correct answer is D. The patients’ symptoms are most consistent with a pheochromocytoma and up to fifty percent of pheochromocytoma’s are due to multiple endocrine neoplasia type 2 (MEN II). Aortic dissection generally does not cause palpitations and might result in a noticeable disparity when comparing BP’s of the upper extremities. A hyperthyroid storm might present similarly however hypothyroidism would not. An insulinoma would be found in MEN type 1 and would result in episodes of hypoglycemia.  Panic attacks are generally a diagnosis of exclusion and should not result in severe blood pressure elevations.

    Multiple Endocrine Neoplasia (MEN) is a hereditary disorder which results in various endocrine gland tumors.  The clinical and laboratory effects vary depending on which glands are most effected. There are 2 types of MEN with the second being divided into 2 subtypes.  MEN type 2 is a genetic mutation of RET gene which causes a protooncogene to convert to an oncogene. Type 2a produces tumors of the medullary thyroid, adrenal gland (pheochromocytoma as described in this case), and parathyroid.  Type IIb results in multiple neuromas (masses of tongue, lips, roof of mouth) instead of parathyroid involvement. MEN Type I involves the parathyroid, pituitary, & pancreas.

    Workup considerations include serum calcitonin, carcinoembryonic antigen, urine and serum metanephrines, parathyroid hormone and calcium.  Metanephrine testing is most specific to the workup of pheochromocytoma which was the most likely cause of the patients’ symptoms in this question. CT, MRI, and/or ultrasound are beneficial in determining the specific location and complexity of a tumor.  Surgical resection is generally recommended in all patients with MEN. First and second-degree family members should be screened for MEN once the diagnosis has been established in a relative.

    References:

    1. https://bestpractice.bmj.com/topics/en-us/866
    2. UpToDate
  12. Question 12 of 30
    12. Question

    A 60-year-old woman reports loss of muscle tone and coordination occurring over the past month.  She has no known past medical history.  On exam, there is a symmetrical decline in muscle tone and eye-hand coordination.  An extensive work-up is unremarkable and you suspect an occult cancer is the most likely explanation.  What test may help confirm the diagnosis?

    Correct

    The correct answer is C. Paraneoplastic antibodies are ordered when a paraneoplastic syndrome is suspected.  Lactic acid is typically used to evaluate for sepsis.  Metanephrine’s are ordered if a pheochromocytoma is likely.  Plasmapheresis is ordered to differentiate between various types of blood disorders.  Procalcitonin is used to differentiate between infections due to bacterial sources as opposed to viral or fungal etiologies.

    Paraneoplastic syndrome is a rare disorder seen mostly in middle aged to older population which occurs when the immune system abnormally responds to cancer-fighting antibodies or T cells that are activated by a malignancy, typically withing the lung, ovaries, lymphatics, or breast.  In most cases, the cancer has not yet been diagnosed and can potentially be so small that it is difficult to detect on imaging. The immune system mistakenly attacks normal cells within the nervous system and can lead to symptoms that mimic a stroke. Symptoms include problems with gait or swallowing, decreased muscle or fine motor control, slurred speech, memory or sleep disturbances, dementia, seizures, or vertigo.

    There are a number of different paraneoplastic antibodies, many of which can help to indicate where the cancer is originating from. Most are detected from the serum but CSF fluid can also be tested. MRI or PET scan can be used to investigate the location of a tumor.

    Treatment is aimed at determining and treating the underlying malignancy first.  There are no cures for paraneoplastic syndrome, but efforts can be directed at decreasing the autoimmune response using steroids, high-dose intravenous immunoglobulin, or irradiation. Therapeutic plasmapheresis can be used in an attempt to remove antibodies from the blood. Functional deficits may improve with speech, physical, and occupational therapies.

     

    References:

    1. https://jamanetwork.com/journals/jamaneurology/fullarticle/774894
    2. https://www.ninds.nih.gov/Disorders/All-Disorders/Paraneoplastic-Syndromes-Information-Page
    3. UpToDate
    Incorrect

    The correct answer is C. Paraneoplastic antibodies are ordered when a paraneoplastic syndrome is suspected.  Lactic acid is typically used to evaluate for sepsis.  Metanephrine’s are ordered if a pheochromocytoma is likely.  Plasmapheresis is ordered to differentiate between various types of blood disorders.  Procalcitonin is used to differentiate between infections due to bacterial sources as opposed to viral or fungal etiologies.

    Paraneoplastic syndrome is a rare disorder seen mostly in middle aged to older population which occurs when the immune system abnormally responds to cancer-fighting antibodies or T cells that are activated by a malignancy, typically withing the lung, ovaries, lymphatics, or breast.  In most cases, the cancer has not yet been diagnosed and can potentially be so small that it is difficult to detect on imaging. The immune system mistakenly attacks normal cells within the nervous system and can lead to symptoms that mimic a stroke. Symptoms include problems with gait or swallowing, decreased muscle or fine motor control, slurred speech, memory or sleep disturbances, dementia, seizures, or vertigo.

    There are a number of different paraneoplastic antibodies, many of which can help to indicate where the cancer is originating from. Most are detected from the serum but CSF fluid can also be tested. MRI or PET scan can be used to investigate the location of a tumor.

    Treatment is aimed at determining and treating the underlying malignancy first.  There are no cures for paraneoplastic syndrome, but efforts can be directed at decreasing the autoimmune response using steroids, high-dose intravenous immunoglobulin, or irradiation. Therapeutic plasmapheresis can be used in an attempt to remove antibodies from the blood. Functional deficits may improve with speech, physical, and occupational therapies.

     

    References:

    1. https://jamanetwork.com/journals/jamaneurology/fullarticle/774894
    2. https://www.ninds.nih.gov/Disorders/All-Disorders/Paraneoplastic-Syndromes-Information-Page
    3. UpToDate
  13. Question 13 of 30
    13. Question

    A tumor in which endocrine gland is responsible for causing Zollinger-Ellison syndrome?

    Correct

    The correct answer is B. Zollinger-Ellison is the result of a pancreatic islet tumor which secrets gastrin, known as a gastrinoma.  The result is gastric or duodenal ulcers which cause pain, bleeding, vomiting, diarrhea, anorexia, and/or weight loss. Another type of pancreatic tumor causes hypersecretion of insulin, known as an insulinoma.

    The most common tumor of the adrenal gland is a catecholamine-secreting tumor in the medulla called a pheochromocytoma.  The classic triad of symptoms includes headache, sweating and tachycardia/palpitations.  Significant elevations in blood pressure occur in approximately 50% of patients. Rarely hypotension, hyperthermia, and confusion can occur. Other adrenal tumors can secrete hormones that cause Cushing’s syndrome or primary aldosteronism.

    Tumors of the parathyroid gland result in hyperparathyroidism and ultimately, hypercalcemia. Hypercalcemia causes the physical manifestation of “bones, stones, abdominal moans, and psychic groans”.  This saying refers to deep bone pains, renal stones, abdominal cramping with constipation, and confusion.  Other symptoms include anorexia, polydipsia, polyuria, and lethargy.

    Pituitary cancer usually manifests with physical symptoms due to the location of the mass.  These include headaches and vision changes.  Some tumors are nonfunctional and result in diminished pituitary function resulting in lower levels of growth hormone (GH), luteinizing hormone (LH), follicle-stimulating hormone (FSH), corticotropin (ACTH), or thyroid-stimulating hormone (TSH). Some cause hypersecretion of FSH or, less commonly, LH.

    Thyroid cancer is most commonly papillary with the next most common being follicular. Symptoms are usually the result of the mass itself. A palpable nodule, dysphagia, hoarse voice, and cervical lymphadenopathy.

     

    References:

    1. UpToDate
    2. https://www.ncbi.nlm.nih.gov/books/NBK9554/
    3. https://my.clevelandclinic.org/health/diseases/12210-thyroid-cancer
    Incorrect

    The correct answer is B. Zollinger-Ellison is the result of a pancreatic islet tumor which secrets gastrin, known as a gastrinoma.  The result is gastric or duodenal ulcers which cause pain, bleeding, vomiting, diarrhea, anorexia, and/or weight loss. Another type of pancreatic tumor causes hypersecretion of insulin, known as an insulinoma.

    The most common tumor of the adrenal gland is a catecholamine-secreting tumor in the medulla called a pheochromocytoma.  The classic triad of symptoms includes headache, sweating and tachycardia/palpitations.  Significant elevations in blood pressure occur in approximately 50% of patients. Rarely hypotension, hyperthermia, and confusion can occur. Other adrenal tumors can secrete hormones that cause Cushing’s syndrome or primary aldosteronism.

    Tumors of the parathyroid gland result in hyperparathyroidism and ultimately, hypercalcemia. Hypercalcemia causes the physical manifestation of “bones, stones, abdominal moans, and psychic groans”.  This saying refers to deep bone pains, renal stones, abdominal cramping with constipation, and confusion.  Other symptoms include anorexia, polydipsia, polyuria, and lethargy.

    Pituitary cancer usually manifests with physical symptoms due to the location of the mass.  These include headaches and vision changes.  Some tumors are nonfunctional and result in diminished pituitary function resulting in lower levels of growth hormone (GH), luteinizing hormone (LH), follicle-stimulating hormone (FSH), corticotropin (ACTH), or thyroid-stimulating hormone (TSH). Some cause hypersecretion of FSH or, less commonly, LH.

    Thyroid cancer is most commonly papillary with the next most common being follicular. Symptoms are usually the result of the mass itself. A palpable nodule, dysphagia, hoarse voice, and cervical lymphadenopathy.

     

    References:

    1. UpToDate
    2. https://www.ncbi.nlm.nih.gov/books/NBK9554/
    3. https://my.clevelandclinic.org/health/diseases/12210-thyroid-cancer
  14. Question 14 of 30
    14. Question

    Which of the following laboratory findings would be present in a patient with syndrome of inappropriate antidiuretic hormone (SIADH)?

    Correct

    The correct answer is A. SIADH is caused by unregulated, increased production, or increased sensitivity of antidiuretic hormone. When this occurs, there is an increase in water permeability at the renal distal tubule and collecting duct. This creates a dilutional hyponatremia which triggers salt excretion. The end result is a euvolaemic hyponatremia. Laboratory studies would show a low serum sodium and serum osmolality as well as elevated urine sodium and osmolality. A BUN to creatinine ration > 20:1 corresponds to prerenal azotemia from dehydration, however, the SIADH patient is euvolaemic.

    SIADH is usually a diagnosis of exclusion and may be triggered by recent surgery, medications, cancer, insults of the brain, or pulmonary disease. The most common prescription drugs that cause SIADH are SSRI’s, however, there are numerous other medications that can cause it as well. Small cell lung cancer is the most common malignancy implicated. Pulmonary etiologies include recent pneumonia, asthma, ARDS and even pnuemothorax. Exogoneous hormone administration, HIV infection, and genetic mutations affecting the renal V2 receptor gene.

    The severity of symptoms due to hyponatremia generally depends on how quickly the sodium drops. If the level has dropped over a relatively short period of time, vomiting, headache, confusion, and seizures are possible due to encephalopathy.  When the sodium level drops more slowly, the symptoms and exam findings can be more subtle.  These can include nausea, fatigue, dizziness, confusion, muscle spasms, gait instability, or memory loss.

    Sodium levels should be corrected slowly in order to avoid osmotic demyelination unless it is known that the hyponatremia occurred over a short period of time. The treatment of SIADH primarily focuses on restricting fluid to 800-1200 mL per day. If the symptoms are severe, hypertonic saline administration is needed. If an underlying cause is determined, it should be addressed if reversible.

     

    References:

    1. https://www.racgp.org.au/download/Documents/AFP/2017/September/AFP-2017-9-Clinical-SIADH.pdf
    2. UpToDate
    3. https://www.ncbi.nlm.nih.gov/books/NBK507777/
    Incorrect

    The correct answer is A. SIADH is caused by unregulated, increased production, or increased sensitivity of antidiuretic hormone. When this occurs, there is an increase in water permeability at the renal distal tubule and collecting duct. This creates a dilutional hyponatremia which triggers salt excretion. The end result is a euvolaemic hyponatremia. Laboratory studies would show a low serum sodium and serum osmolality as well as elevated urine sodium and osmolality. A BUN to creatinine ration > 20:1 corresponds to prerenal azotemia from dehydration, however, the SIADH patient is euvolaemic.

    SIADH is usually a diagnosis of exclusion and may be triggered by recent surgery, medications, cancer, insults of the brain, or pulmonary disease. The most common prescription drugs that cause SIADH are SSRI’s, however, there are numerous other medications that can cause it as well. Small cell lung cancer is the most common malignancy implicated. Pulmonary etiologies include recent pneumonia, asthma, ARDS and even pnuemothorax. Exogoneous hormone administration, HIV infection, and genetic mutations affecting the renal V2 receptor gene.

    The severity of symptoms due to hyponatremia generally depends on how quickly the sodium drops. If the level has dropped over a relatively short period of time, vomiting, headache, confusion, and seizures are possible due to encephalopathy.  When the sodium level drops more slowly, the symptoms and exam findings can be more subtle.  These can include nausea, fatigue, dizziness, confusion, muscle spasms, gait instability, or memory loss.

    Sodium levels should be corrected slowly in order to avoid osmotic demyelination unless it is known that the hyponatremia occurred over a short period of time. The treatment of SIADH primarily focuses on restricting fluid to 800-1200 mL per day. If the symptoms are severe, hypertonic saline administration is needed. If an underlying cause is determined, it should be addressed if reversible.

     

    References:

    1. https://www.racgp.org.au/download/Documents/AFP/2017/September/AFP-2017-9-Clinical-SIADH.pdf
    2. UpToDate
    3. https://www.ncbi.nlm.nih.gov/books/NBK507777/
  15. Question 15 of 30
    15. Question

    Which of the following is the most common symptom that a patient with new onset hyperparathyroidism would be experiencing?

    Correct

    The correct answer is B. The vast majority of patients are asymptomatic at the time of diagnosis.  Hypercalcemia on routine blood work is the usual scenario leading to the diagnosis. The other symptoms listed are all possible with bone pain, renal stones, and weakness or fatigue being the most common in that order. In addition, nausea, anorexia, depression and confusion are also possible.

    Causes of hyperparathyroidism can be divided into 2 categories: primary and secondary. Primary causes include parathyroid cancer (adenoma accounts for up to 85% of all cases), radiation exposure to the head or neck, and genetic defects. Secondary causes include chronic kidney disease, low calcium dietary intake, calcium malabsorption (bariatric surgery, celiac or pancreatic disease), renal calcium losses, and inhibition of bone resorption. Women over age 60 are the most common demographic affected.

    Once hypercalcemia has been detected, work-up includes labs and imaging to determine the underlying cause and look for evidence of systemic effects.  Lab tests should include PTH, alkaline phophatase, phosphorus, and 24-hou urine. A bone mineral density (DXA) test can be ordered to assess for fracture or softening.  Ultrasound can detect the presence of stones within the kidneys as well as masses within the parathyroid gland.  A nuclear scan can also be helpful in the workup of benign tumors.

    Surgery parathyroidectomy is the standard treatment of choice. If contraindicated, preventative measures should be taken to avoid hypercalcemia.  Avoid using thiazide diuretics and excess dietary calcium intake. Encourage exercise to minimize bone resorption and moderate vitamin D intake.

    References:

    1. https://medlineplus.gov/ency/article/001215.htm
    2. UpToDate
    Incorrect

    The correct answer is B. The vast majority of patients are asymptomatic at the time of diagnosis.  Hypercalcemia on routine blood work is the usual scenario leading to the diagnosis. The other symptoms listed are all possible with bone pain, renal stones, and weakness or fatigue being the most common in that order. In addition, nausea, anorexia, depression and confusion are also possible.

    Causes of hyperparathyroidism can be divided into 2 categories: primary and secondary. Primary causes include parathyroid cancer (adenoma accounts for up to 85% of all cases), radiation exposure to the head or neck, and genetic defects. Secondary causes include chronic kidney disease, low calcium dietary intake, calcium malabsorption (bariatric surgery, celiac or pancreatic disease), renal calcium losses, and inhibition of bone resorption. Women over age 60 are the most common demographic affected.

    Once hypercalcemia has been detected, work-up includes labs and imaging to determine the underlying cause and look for evidence of systemic effects.  Lab tests should include PTH, alkaline phophatase, phosphorus, and 24-hou urine. A bone mineral density (DXA) test can be ordered to assess for fracture or softening.  Ultrasound can detect the presence of stones within the kidneys as well as masses within the parathyroid gland.  A nuclear scan can also be helpful in the workup of benign tumors.

    Surgery parathyroidectomy is the standard treatment of choice. If contraindicated, preventative measures should be taken to avoid hypercalcemia.  Avoid using thiazide diuretics and excess dietary calcium intake. Encourage exercise to minimize bone resorption and moderate vitamin D intake.

    References:

    1. https://medlineplus.gov/ency/article/001215.htm
    2. UpToDate
  16. Question 16 of 30
    16. Question

    Which of the following is the most common etiology of hypoparathyroidism?

    Correct

    The correct answer is E. All disorders listed can cause hypoparathyroidism. However, hypoparathyroidism is most commonly the result of surgical excision which may be transient (recovery in days, weeks, or months), intermittent, or permanent. Transient occurred in up to 20% of patients after surgery for thyroid cancer, and permanent hypoparathyroidism in up to 3.0% of patients after total thyroidectomy. Transient hypoparathyroidism may be due to manipulation of the blood supply to or removal of one or more parathyroid glands during surgery, whereas intermittent hypoparathyroidism is due to decreased parathyroid reserve (parathyroid insufficiency).

    Symptoms and exam findings of hypoparathyroidism are due to the resulting hypocalcemia which causes neuromuscular irritability. Symptoms include perioral numbness, paresthesias, muscle cramps, carpopedal spasm, laryngospasm, and even seizures. The classic physical exam findings are a positive Trousseau’s or Chvostek’s sign. Trousseau’s sign is assessed by inflating a blood pressure cuff above the patients’ systolic blood pressure for three minutes while assessing for carpal spasms of the fingers or wrist. Chvostek’s sign is elicited by tapping along the course of the facial nerve just anterior to the ear and observing for lip twitching or overt spasm of the facial muscles on the same side. These findings can occur in approximately 10% of patients without hypocalcemia. Other effects of hypocalcemia include hypotension, prolonged QT interval, and emotional liability. Patients with chronic hypoparathyroidism and poorly managed hypocalcemia can develop dementia, parkinsonism, cataracts, poor dentition, and dry skin.

    The primary laboratory findings are hypocalcemia and a low serum intact parathyroid hormone (PTH) level. Most patients will also have an elevated serum phosphorus level. Treatment of acute hypocalcemia includes IV calcium gluconate infused over 10 to 20 minutes, followed by oral calcitrol and calcium carbonate. If magnesium is low, that should be corrected prior to initiating calcium therapy.  Cases of chronic hypoparathyroidism may benefit from subcutaneous administration of recombinant PTH 1-34 and 1-84 hormones in order to minimize the need for supplemental calcium therapy.

     

    References:

    1. UpToDate
    2. https://jamanetwork.com/journals/jamasurgery/fullarticle/2542667
    Incorrect

    The correct answer is E. All disorders listed can cause hypoparathyroidism. However, hypoparathyroidism is most commonly the result of surgical excision which may be transient (recovery in days, weeks, or months), intermittent, or permanent. Transient occurred in up to 20% of patients after surgery for thyroid cancer, and permanent hypoparathyroidism in up to 3.0% of patients after total thyroidectomy. Transient hypoparathyroidism may be due to manipulation of the blood supply to or removal of one or more parathyroid glands during surgery, whereas intermittent hypoparathyroidism is due to decreased parathyroid reserve (parathyroid insufficiency).

    Symptoms and exam findings of hypoparathyroidism are due to the resulting hypocalcemia which causes neuromuscular irritability. Symptoms include perioral numbness, paresthesias, muscle cramps, carpopedal spasm, laryngospasm, and even seizures. The classic physical exam findings are a positive Trousseau’s or Chvostek’s sign. Trousseau’s sign is assessed by inflating a blood pressure cuff above the patients’ systolic blood pressure for three minutes while assessing for carpal spasms of the fingers or wrist. Chvostek’s sign is elicited by tapping along the course of the facial nerve just anterior to the ear and observing for lip twitching or overt spasm of the facial muscles on the same side. These findings can occur in approximately 10% of patients without hypocalcemia. Other effects of hypocalcemia include hypotension, prolonged QT interval, and emotional liability. Patients with chronic hypoparathyroidism and poorly managed hypocalcemia can develop dementia, parkinsonism, cataracts, poor dentition, and dry skin.

    The primary laboratory findings are hypocalcemia and a low serum intact parathyroid hormone (PTH) level. Most patients will also have an elevated serum phosphorus level. Treatment of acute hypocalcemia includes IV calcium gluconate infused over 10 to 20 minutes, followed by oral calcitrol and calcium carbonate. If magnesium is low, that should be corrected prior to initiating calcium therapy.  Cases of chronic hypoparathyroidism may benefit from subcutaneous administration of recombinant PTH 1-34 and 1-84 hormones in order to minimize the need for supplemental calcium therapy.

     

    References:

    1. UpToDate
    2. https://jamanetwork.com/journals/jamasurgery/fullarticle/2542667
  17. Question 17 of 30
    17. Question


    A pediatric patient you are seeing has the bold lines on the attached growth chart.  In addition to prolactin, what other screening test will most likely be elevated?

    Correct

    The correct answer is C. Insulin-like growth factor is used in the diagnosis of gigantism and acromegaly, which is indicated by the significantly steep height line. The gold standard test is the growth hormone suppression test. The plasma corticotropin (ACTH) and cortisol tests are used in the diagnosis of adrenal insufficiency (Cushing’s syndrome). Metanephrine is used in the diagnosis of pheochromocytoma. And testosterone is used in suspected hypogonadism.

    Gigantism and acromegaly are genetic disorders resulting in excess growth hormone and insulin-like growth factor. Gigantism is characterized by tall stature resulting from these excess hormones occurring prior to fusion of the growth plates while acromegaly is characterized by large hands and feet as well as coarse facial features occurring after the fusion of the growth plates. These elevated levels lead to rapid, excessive linear growth and an extremely tall child or adult if left untreated. It is also usually associated with weight gain as well (also indicated on the attached growth chart). The diagnosis should be suspected when the patient’s height is 3 standard deviations above normal mean height or 2 standard deviations above the adjusted mean parental height. In adolescent females, additional symptoms include amenorrhea with or without galactorrhea. If the excess growth hormone occurs in adults, the effects are usually minimal.  Findings more specific to acromegaly include hyperhidrosis, skin tags, weakness, lethargy, and carpal tunnel syndrome. Facies changes also occur but can be more subtle and go unnoticed. Malocclusion and temporomandibular joint pains may be noticed. If a pituitary growth hormone secreting tumor is large enough, it can result in symptoms related to encroachment of the surrounding structures leading to visual field deficits, ophthalmoplegia, and headaches.

    Treatment is case specific. Surgical resection or radiation are options in addition to pharmacologic options.  These include bromocriptine, a dopamine analog, octreotide, a somatostatin analog, lanreotide, a sustained-release formulation of a somatostatin analog, and pegvisomant, a genetically engineered competitive GH receptor antagonist. Prognosis is good when recognized and treated early. If untreated, acromegaly has increased mortality as compared to the general population due to resulting cardiovascular or respiratory complications and malignancies.

     

    References:

    1. https://www.ncbi.nlm.nih.gov/books/NBK538261/
    2. UpToDate
    Incorrect

    The correct answer is C. Insulin-like growth factor is used in the diagnosis of gigantism and acromegaly, which is indicated by the significantly steep height line. The gold standard test is the growth hormone suppression test. The plasma corticotropin (ACTH) and cortisol tests are used in the diagnosis of adrenal insufficiency (Cushing’s syndrome). Metanephrine is used in the diagnosis of pheochromocytoma. And testosterone is used in suspected hypogonadism.

    Gigantism and acromegaly are genetic disorders resulting in excess growth hormone and insulin-like growth factor. Gigantism is characterized by tall stature resulting from these excess hormones occurring prior to fusion of the growth plates while acromegaly is characterized by large hands and feet as well as coarse facial features occurring after the fusion of the growth plates. These elevated levels lead to rapid, excessive linear growth and an extremely tall child or adult if left untreated. It is also usually associated with weight gain as well (also indicated on the attached growth chart). The diagnosis should be suspected when the patient’s height is 3 standard deviations above normal mean height or 2 standard deviations above the adjusted mean parental height. In adolescent females, additional symptoms include amenorrhea with or without galactorrhea. If the excess growth hormone occurs in adults, the effects are usually minimal.  Findings more specific to acromegaly include hyperhidrosis, skin tags, weakness, lethargy, and carpal tunnel syndrome. Facies changes also occur but can be more subtle and go unnoticed. Malocclusion and temporomandibular joint pains may be noticed. If a pituitary growth hormone secreting tumor is large enough, it can result in symptoms related to encroachment of the surrounding structures leading to visual field deficits, ophthalmoplegia, and headaches.

    Treatment is case specific. Surgical resection or radiation are options in addition to pharmacologic options.  These include bromocriptine, a dopamine analog, octreotide, a somatostatin analog, lanreotide, a sustained-release formulation of a somatostatin analog, and pegvisomant, a genetically engineered competitive GH receptor antagonist. Prognosis is good when recognized and treated early. If untreated, acromegaly has increased mortality as compared to the general population due to resulting cardiovascular or respiratory complications and malignancies.

     

    References:

    1. https://www.ncbi.nlm.nih.gov/books/NBK538261/
    2. UpToDate
  18. Question 18 of 30
    18. Question

     

    A 10-year-old boy presents with urinary frequency and increased thirst. Labs have been obtained (see below).  Which of the following would be most useful in the initial work-up of this patient?

    Correct

    The correct answer is D. This patient is exhibiting symptoms and initial lab testing consistent with diabetes insipidus. Hemoglobin A1C is used in the work-up and management of diabetes mellitus. There is no indication for a renal biopsy at this point.

    Diabetes insipidus is caused by either a decreased release or response to antidiuretic hormone (ADH) resulting in elevations of serum sodium and osmolality. Most cases are diagnoses between the age of 10-20. There are two types of diabetes insipidus, central and nephrogenic, and each has congenital and acquired causes. The most common etiologies include idiopathic, tumors, infiltrative diseases, neurosurgery, and trauma. Congenital disorders are also possibilities and patients with anorexia can also develop mild cases. Most patients will present reporting polyuria, polydipsia, and nocturia. Other symptoms may include constipation, vomiting, fevers, fussiness, and delayed growth. If a brain tumor is the cause, headaches, and visual defects may present. There may be evidence of dehydration on physical exam.

    A water deprivation test accompanied by desmopressin (DDAVP) is used to differentiate central versus nephrogenic diabetes insipidus. Typically, a 7-hour deprivation test is adequate. Deprivation of water intake should result in a concentrated urine if primary polydipsia is the cause. Adding desmopressin after water deprivation should result in concentrated urine in those with central DI while no significant response should occur in cases of nephrogenic DI. The treatment for central DI is usually DDAVP until the disorder has resolved. Monitor for hyponatremia during treatment. Nephrogenic DI is primarily treated by addressing the underlying cause however thiazide diuretics, DDAVP and even NSAIDs can be used in addition to a low-solute diet.

    With treatment and addressing the underlying cause, if present, the prognosis of DI is very good. More severe symptoms occur if the disorder goes undiagnosed and if the patient does not have adequate access to water.

    References:

    1. https://www.ncbi.nlm.nih.gov/books/NBK470458/
    2. UpToDate
    3. https://www.ncbi.nlm.nih.gov/books/NBK537591/
    Incorrect

    The correct answer is D. This patient is exhibiting symptoms and initial lab testing consistent with diabetes insipidus. Hemoglobin A1C is used in the work-up and management of diabetes mellitus. There is no indication for a renal biopsy at this point.

    Diabetes insipidus is caused by either a decreased release or response to antidiuretic hormone (ADH) resulting in elevations of serum sodium and osmolality. Most cases are diagnoses between the age of 10-20. There are two types of diabetes insipidus, central and nephrogenic, and each has congenital and acquired causes. The most common etiologies include idiopathic, tumors, infiltrative diseases, neurosurgery, and trauma. Congenital disorders are also possibilities and patients with anorexia can also develop mild cases. Most patients will present reporting polyuria, polydipsia, and nocturia. Other symptoms may include constipation, vomiting, fevers, fussiness, and delayed growth. If a brain tumor is the cause, headaches, and visual defects may present. There may be evidence of dehydration on physical exam.

    A water deprivation test accompanied by desmopressin (DDAVP) is used to differentiate central versus nephrogenic diabetes insipidus. Typically, a 7-hour deprivation test is adequate. Deprivation of water intake should result in a concentrated urine if primary polydipsia is the cause. Adding desmopressin after water deprivation should result in concentrated urine in those with central DI while no significant response should occur in cases of nephrogenic DI. The treatment for central DI is usually DDAVP until the disorder has resolved. Monitor for hyponatremia during treatment. Nephrogenic DI is primarily treated by addressing the underlying cause however thiazide diuretics, DDAVP and even NSAIDs can be used in addition to a low-solute diet.

    With treatment and addressing the underlying cause, if present, the prognosis of DI is very good. More severe symptoms occur if the disorder goes undiagnosed and if the patient does not have adequate access to water.

    References:

    1. https://www.ncbi.nlm.nih.gov/books/NBK470458/
    2. UpToDate
    3. https://www.ncbi.nlm.nih.gov/books/NBK537591/
  19. Question 19 of 30
    19. Question

    What is the most effective treatment option for a patient with achondroplasia?

    Correct

    The correct answer is B. Achondroplasia is the most common cause of dwarfism. Growth hormone is not advised and could potentially worsen the disorder in cases of achondroplasia but may be beneficial in some other etiologies. Limb lengthening surgeries have been performed for this in the past and are still performed periodically to this day but are controversial. Testosterone is used for cases of hypogonadism but not for dwarfism. Vitamin D is used for another skeletal disorder, ricketts.

    Dwarfism (short stature < 4’10”) is usually the result of any of over 300 primary conditions of which achondroplasia is the most common.  Other common conditions include various genetic disorders, malnutrition, and kidney disease, as well as metabolism and hormone disorders. It most often does occur in families where both parents are of average height. Achondroplasia is an autosomal dominant condition resulting from variants in the fibroblast growth factor receptor 3 (FGFR3) gene.

    Karyotyping genetic testing can be performed as well as testing to exclude other disorders as an underlying cause. These additional tests include, complete blood count, sweat chloride test for cystic fibrosis, thyrotropin (TSH) levels and free thyroxine levels (T4), sedimentation rate, antibody testing for celiac sprue, and pre-albumin and transferrin. Useful imaging can include x-ray’s of the hand and wrist to predict the bone age and forecast adult height, and MRI to assess for intracranial masses.

    Treatment is aimed at the underlying disorder which may or may not have a specific treatment. Recombinant human growth hormone therapy is effective for patients suffering from growth hormone deficiency. Gonadotropin-releasing hormone analogs are used to halt the progression of precocious puberty. Supportive care includes the involvement of both physical and occupational therapists as well as focusing on weight management, otitis media prevention, and sleep apnea management. Most patients with dwarfism have a normal life expectancy and quality of life can be improved with the supportive care measures listed above.

    References:

    1. https://rarediseases.info.nih.gov/diseases/1988/dwarfism
    2. https://www.ncbi.nlm.nih.gov/books/NBK563282/
    3. UpToDate
    Incorrect

    The correct answer is B. Achondroplasia is the most common cause of dwarfism. Growth hormone is not advised and could potentially worsen the disorder in cases of achondroplasia but may be beneficial in some other etiologies. Limb lengthening surgeries have been performed for this in the past and are still performed periodically to this day but are controversial. Testosterone is used for cases of hypogonadism but not for dwarfism. Vitamin D is used for another skeletal disorder, ricketts.

    Dwarfism (short stature < 4’10”) is usually the result of any of over 300 primary conditions of which achondroplasia is the most common.  Other common conditions include various genetic disorders, malnutrition, and kidney disease, as well as metabolism and hormone disorders. It most often does occur in families where both parents are of average height. Achondroplasia is an autosomal dominant condition resulting from variants in the fibroblast growth factor receptor 3 (FGFR3) gene.

    Karyotyping genetic testing can be performed as well as testing to exclude other disorders as an underlying cause. These additional tests include, complete blood count, sweat chloride test for cystic fibrosis, thyrotropin (TSH) levels and free thyroxine levels (T4), sedimentation rate, antibody testing for celiac sprue, and pre-albumin and transferrin. Useful imaging can include x-ray’s of the hand and wrist to predict the bone age and forecast adult height, and MRI to assess for intracranial masses.

    Treatment is aimed at the underlying disorder which may or may not have a specific treatment. Recombinant human growth hormone therapy is effective for patients suffering from growth hormone deficiency. Gonadotropin-releasing hormone analogs are used to halt the progression of precocious puberty. Supportive care includes the involvement of both physical and occupational therapists as well as focusing on weight management, otitis media prevention, and sleep apnea management. Most patients with dwarfism have a normal life expectancy and quality of life can be improved with the supportive care measures listed above.

    References:

    1. https://rarediseases.info.nih.gov/diseases/1988/dwarfism
    2. https://www.ncbi.nlm.nih.gov/books/NBK563282/
    3. UpToDate
  20. Question 20 of 30
    20. Question

    What is the most common symptom of a patient presenting with a pituitary adenoma?

    Correct

    The correct answer is A. The mass occurring in the optic chiasm leads to vision loss with both temporal fields being involved being the most common findings but one or both eyes may be affected to variable degrees. These visual defects can occur so gradually that many patients do not appreciate the loss for months or years. Headaches and diplopia also occur but to a lesser degree. Ocular muscle paralysis is also possible.  The mass usually does not result in diffuse paresthesias or tinnitus.

    Pituitary adenomas can arise from any type of cell and may cause an increased secretion of the hormone(s) produced by that cell and/or decreased secretion of other hormones resulting from compression.  These masses are classified by the size and cell of origin. If less than 1 cm, they are termed microadenomas. If greater than 1 cm they are known as macroadenomas. Cysts, abscesses, arteriovenous malformations, and hyperplasia can also present with the same symptoms.

    The primary modality used in the work-up is the MRI with contrast. A CT scan may show calcifications which should result in ordering a follow-up MRI for additional detail of the area. Hormonal evaluation is also performed.  This includes prolactin, insulin-like growth factor-1, plasma corticotropin, 24-hour urinary free cortisol, luteinizing hormone, follicle-stimulating hormone, total or free thyroxine, and thyroid-stimulating hormone.

    Treatment can involve medications to reduce hormone hypersecretion, neurosurgical resection, and or radiation to reduce the size. Dopamine agonists inhibit the release of prolactin from the anterior pituitary which can help with hyperprolactinemia symptoms, shrink tumor size, and restore reproductive function. Somatostatin analogues are used in the management of growth hormone– and ACTH-secreting tumors. If the mass is a nonfunctioning microadenoma, annual reimaging and monitoring can be used.

     

    References:

    1. UpToDate
    2. https://www.aafp.org/afp/2013/0901/p319.html
    Incorrect

    The correct answer is A. The mass occurring in the optic chiasm leads to vision loss with both temporal fields being involved being the most common findings but one or both eyes may be affected to variable degrees. These visual defects can occur so gradually that many patients do not appreciate the loss for months or years. Headaches and diplopia also occur but to a lesser degree. Ocular muscle paralysis is also possible.  The mass usually does not result in diffuse paresthesias or tinnitus.

    Pituitary adenomas can arise from any type of cell and may cause an increased secretion of the hormone(s) produced by that cell and/or decreased secretion of other hormones resulting from compression.  These masses are classified by the size and cell of origin. If less than 1 cm, they are termed microadenomas. If greater than 1 cm they are known as macroadenomas. Cysts, abscesses, arteriovenous malformations, and hyperplasia can also present with the same symptoms.

    The primary modality used in the work-up is the MRI with contrast. A CT scan may show calcifications which should result in ordering a follow-up MRI for additional detail of the area. Hormonal evaluation is also performed.  This includes prolactin, insulin-like growth factor-1, plasma corticotropin, 24-hour urinary free cortisol, luteinizing hormone, follicle-stimulating hormone, total or free thyroxine, and thyroid-stimulating hormone.

    Treatment can involve medications to reduce hormone hypersecretion, neurosurgical resection, and or radiation to reduce the size. Dopamine agonists inhibit the release of prolactin from the anterior pituitary which can help with hyperprolactinemia symptoms, shrink tumor size, and restore reproductive function. Somatostatin analogues are used in the management of growth hormone– and ACTH-secreting tumors. If the mass is a nonfunctioning microadenoma, annual reimaging and monitoring can be used.

     

    References:

    1. UpToDate
    2. https://www.aafp.org/afp/2013/0901/p319.html
  21. Question 21 of 30
    21. Question

    A patient presents for evaluation of palpitations and difficulty feeling cool during the hot, summer months. Review of systems is positive for unintentional weight loss despite feeling hungry all the time.  What is likely to be found on physical exam based on the most likely diagnosis?

    Correct

    The correct answer is D. The patient presents with classic symptoms of hyperthyroidism. Most patients with hyperthyroidism will have some degree of a goiter which they may or may not be aware of. Hyperthyroidism also usually results in tachycardia, warm skin, and possible conjunctival edema (chemosis).

    Other symptoms and physical finding of hyperthyroidism can include:

    skin – smooth texture, loosening of the nails from the nail bed or softening, hyperpigmentation,

    pruritus, and thin hair

    eyes – lid lag, exophthalmos, conjunctival edema, or gritty sensation

    cardiac – hypertension, cardiomyopathy, heart failure, and atrial fibrillation

    respiratory – dyspnea, asthma exacerbation

    urinary – polyuria and nocturia

    psychiatric – psychosis, agitation, and depression

    The most common cause of hyperthyroidism is Graves’ disease, with other underlying causes including toxic nodular goiter, thyroiditis, and iodine or drug-induced thyroid dysfunction. The disease is more common in women, white race, and advances in age. Hyperthyroidism can be symptomatic or subclinical. Symptomatic is associated with a low serum thyroid-stimulating hormone (TSH) concentrations and elevations of thyroxine (T4) and/or tri-iodothyronine (T3). Subclinical hyperthyroidism is associated with low serum TSH, but normal serum T4 and T3 concentrations.

     

    Work-up starts a serum TSH followed by a free T4 or free T4 index, and free or total T3 if the TSH is low. A thyroid radioactive iodine uptake test or thyroid ultrasound are also usually ordered to look for various causes of hyperthyroidism. Medications, radioactive iodine and surgery are the treatment options. Thiamazole as the preferred medication for treatment. Beta blockers are used in the treatment of symptoms (palpitations, tachycardia, tremulousness, anxiety, heat intolerance) from the time of inital diagnosis until resolution of hyperthyroidism via one of the definitive treatment options. Radioactive iodine therapy can also be considered as first-line treatment for Graves’ disease, toxic adenoma, and toxic multinodular goiter, but should be avoided if cancer is suspected or confirmed on biopsy. Total thyroidectomy is recommended in cases of cancer and is the most effective treatment for Graves’ disease.

     

    References:

    1. https://www.ncbi.nlm.nih.gov/pmc/articles/PMC5014602/
    2. UpToDate
    3. https://www.aafp.org/afp/2016/0301/p363.html
    Incorrect

    The correct answer is D. The patient presents with classic symptoms of hyperthyroidism. Most patients with hyperthyroidism will have some degree of a goiter which they may or may not be aware of. Hyperthyroidism also usually results in tachycardia, warm skin, and possible conjunctival edema (chemosis).

    Other symptoms and physical finding of hyperthyroidism can include:

    skin – smooth texture, loosening of the nails from the nail bed or softening, hyperpigmentation,

    pruritus, and thin hair

    eyes – lid lag, exophthalmos, conjunctival edema, or gritty sensation

    cardiac – hypertension, cardiomyopathy, heart failure, and atrial fibrillation

    respiratory – dyspnea, asthma exacerbation

    urinary – polyuria and nocturia

    psychiatric – psychosis, agitation, and depression

    The most common cause of hyperthyroidism is Graves’ disease, with other underlying causes including toxic nodular goiter, thyroiditis, and iodine or drug-induced thyroid dysfunction. The disease is more common in women, white race, and advances in age. Hyperthyroidism can be symptomatic or subclinical. Symptomatic is associated with a low serum thyroid-stimulating hormone (TSH) concentrations and elevations of thyroxine (T4) and/or tri-iodothyronine (T3). Subclinical hyperthyroidism is associated with low serum TSH, but normal serum T4 and T3 concentrations.

     

    Work-up starts a serum TSH followed by a free T4 or free T4 index, and free or total T3 if the TSH is low. A thyroid radioactive iodine uptake test or thyroid ultrasound are also usually ordered to look for various causes of hyperthyroidism. Medications, radioactive iodine and surgery are the treatment options. Thiamazole as the preferred medication for treatment. Beta blockers are used in the treatment of symptoms (palpitations, tachycardia, tremulousness, anxiety, heat intolerance) from the time of inital diagnosis until resolution of hyperthyroidism via one of the definitive treatment options. Radioactive iodine therapy can also be considered as first-line treatment for Graves’ disease, toxic adenoma, and toxic multinodular goiter, but should be avoided if cancer is suspected or confirmed on biopsy. Total thyroidectomy is recommended in cases of cancer and is the most effective treatment for Graves’ disease.

     

    References:

    1. https://www.ncbi.nlm.nih.gov/pmc/articles/PMC5014602/
    2. UpToDate
    3. https://www.aafp.org/afp/2016/0301/p363.html
  22. Question 22 of 30
    22. Question

    Which test is most specific to making the diagnosis of Graves’ disease?

    Correct

    The correct answer is E.  Graves’ disease is caused by excess thyroid stimulating immunoglobulin which is generated from within the thyroid cells, lymph nodes, or bone marrow. Thyroid stimulating immunoglobulin binds with thyroid-stimulating hormone (TSH) receptor on the thyroid cell membrane and stimulates the action of the thyroid-stimulating hormone. The result is a hyperthyroid state and thyromegaly. The other tests listed are also utilized in the initial work-up of suspected hyperthyroidism but are not specific to Graves’. A T3/T4 ratio > 20 or FT3/FT4 ratio > than 0.3 suggests Graves’ disease as opposed to other etiologies of hyperthyroidism such as thyrotoxicosis or thyroiditis.

    Graves’ disease is responsible for approximately 60-80% of cases of hyperthyroidism. The disorder can be genetic or the result of an initial trigger such as postpartum pregnancy, infection, emotional stress, smoking, or exogenous iodine which triggers an immune response on susceptible genes. The hallmark findings in Graves’ disease are hyperthyroidism, goiter, and thyroid eye disease (orbitopathy). Any symptoms or exam findings associated with hyperthyroidism are also possible. Graves’ orbitopathy is the result of inflammation, cellular proliferation and excess growth of the extraocular muscles, connective tissue, and adipose tissues resulting in proptosis, diplopia, congestion, and periorbital edema.

    Additional testing can include a radioactive iodine uptake scan which will show diffuse increased uptake. A toxic nodule causes focal uptake known as a hot nodule, while a toxic multinodular goiter will have heterogeneous uptake and low uptake occurs in cases of silent thyroiditis or factitious hyperthyroidism. An ultrasound with Doppler will demonstrate a hypervascular thyroid gland.

    Treatment is aimed at urgent symptom management while initiating the process of correcting the underlying problem. A beta blocker such as atenolol 25-50 mg orally once daily or propranolol 10-40 mg orally every six to eight hours is preferred but calcium channel blockers like diltiazem and verapamil can be used if beta blockers are contraindicated. Prednisone can be used to reduce orbitopathy. Addressing the underlying cause can be accomplished by antithyroid drugs, radioactive iodine treatment of the thyroid gland, or total/subtotal thyroidectomy. The antithyroid drug of choice is methimazole with dosing being based on how high above the upper limit of normal the FT4 is.

     

    References:

    1. https://www.ncbi.nlm.nih.gov/books/NBK448195/
    2. UpToDate
    Incorrect

    The correct answer is E.  Graves’ disease is caused by excess thyroid stimulating immunoglobulin which is generated from within the thyroid cells, lymph nodes, or bone marrow. Thyroid stimulating immunoglobulin binds with thyroid-stimulating hormone (TSH) receptor on the thyroid cell membrane and stimulates the action of the thyroid-stimulating hormone. The result is a hyperthyroid state and thyromegaly. The other tests listed are also utilized in the initial work-up of suspected hyperthyroidism but are not specific to Graves’. A T3/T4 ratio > 20 or FT3/FT4 ratio > than 0.3 suggests Graves’ disease as opposed to other etiologies of hyperthyroidism such as thyrotoxicosis or thyroiditis.

    Graves’ disease is responsible for approximately 60-80% of cases of hyperthyroidism. The disorder can be genetic or the result of an initial trigger such as postpartum pregnancy, infection, emotional stress, smoking, or exogenous iodine which triggers an immune response on susceptible genes. The hallmark findings in Graves’ disease are hyperthyroidism, goiter, and thyroid eye disease (orbitopathy). Any symptoms or exam findings associated with hyperthyroidism are also possible. Graves’ orbitopathy is the result of inflammation, cellular proliferation and excess growth of the extraocular muscles, connective tissue, and adipose tissues resulting in proptosis, diplopia, congestion, and periorbital edema.

    Additional testing can include a radioactive iodine uptake scan which will show diffuse increased uptake. A toxic nodule causes focal uptake known as a hot nodule, while a toxic multinodular goiter will have heterogeneous uptake and low uptake occurs in cases of silent thyroiditis or factitious hyperthyroidism. An ultrasound with Doppler will demonstrate a hypervascular thyroid gland.

    Treatment is aimed at urgent symptom management while initiating the process of correcting the underlying problem. A beta blocker such as atenolol 25-50 mg orally once daily or propranolol 10-40 mg orally every six to eight hours is preferred but calcium channel blockers like diltiazem and verapamil can be used if beta blockers are contraindicated. Prednisone can be used to reduce orbitopathy. Addressing the underlying cause can be accomplished by antithyroid drugs, radioactive iodine treatment of the thyroid gland, or total/subtotal thyroidectomy. The antithyroid drug of choice is methimazole with dosing being based on how high above the upper limit of normal the FT4 is.

     

    References:

    1. https://www.ncbi.nlm.nih.gov/books/NBK448195/
    2. UpToDate
  23. Question 23 of 30
    23. Question

    Classic symptoms of fatigue, cold intolerance, and dry skin are most commonly associated with which disorder?

    Correct

    The correct answer is D. Addison’s is associated with hyperpigmentation (bronzing), hypotension and GI symptoms. Cushing is associated with moon faces, buffalo hump, and hirsutism. Hypothyroidism is associated with cold intolerance, palpitations, and unintentional weight loss.  Lupus symptoms include a butterfly rash and arthralgias.

    Hypothyroidism is more prevalent in women, elderly, and white individuals with causes being classified as either primary (thyroid gland unable to produce thyroid hormone) or secondary (TSH deficiency related to dysfunction of the pituitary or hypothalamus) or tertiary (caused by any disorder that damages the hypothalamus or interferes with hypothalamic-pituitary portal blood flow). Etiologies of primary hypothyroidism include autoimmune thyroiditis, dietary iodine deficiency, thyroidectomy, radioiodine therapy for hyperthyroidism, neck radiation, medications, and infiltrative diseases. Secondary hypothyroidism is much less common, but etiologies include pituitary tumor, postpartum pituitary necrosis (Sheehan syndrome), trauma, hypophysitis, nonpituitary tumors, and inactivating mutations in the gene for either TSH or the TSH receptor.

    In addition to the findings mentioned above, additional patient complaints may include voice changes, hair loss, constipation, muscle cramps, sleep disturbances, menstrual cycle abnormalities, weight gain, depression/anxiety, psychosis, and cognitive dysfunction . Findings on physical exam can include enlarged thyroid gland, slow speech/movements or dull facial expressions, coarse and brittle hair, pallor, and bradycardia.

    Work-up starts a TSH level which is usually elevated and a free T4 level which is usually low. Subclinical cases may have a normal free T4. If suspected, thyroid peroxidase antibodies should be assessed. Additional lab abnormalities which could be present if tested include hyperlipidemia, anemia, and elevations of CK, liver enzymes, BUN, creatinine, or uric acid. Unlike with hyperthyroidism, ultrasound is usually not indicated for hypothyroidism. Asymptomatic screening should be considered in patients with the following risk factors: significant hyperlipidemia, hyponatremia, macrocytic anemia, pericardial or pleural effusions, prior thyroid injury, pituitary or hypothalamic disorders, and a history of autoimmune disorders.

    The primary treatment of hypothyroidism is to give synthetic thyroxine (T4, levothyroxine) with the average adult dosage being 1.6 mcg/kg/day taken ideally 30-60 minutes prior to breakfast. Elderly or patients with coronary artery disease may benefit from starting at a lower dose while obese patients may ultimately require higher doses. A TSH should be redrawn in 4-6 weeks and the dose of T4 increased by 12 to 25 mcg/day every 3-6 weeks as needed until the TSH returns to the normal reference level.

    References:

    1. UpToDate
    2. https://www.ncbi.nlm.nih.gov/pmc/articles/PMC6619426/
    3. https://www.ncbi.nlm.nih.gov/books/NBK519536/
    Incorrect

    The correct answer is D. Addison’s is associated with hyperpigmentation (bronzing), hypotension and GI symptoms. Cushing is associated with moon faces, buffalo hump, and hirsutism. Hypothyroidism is associated with cold intolerance, palpitations, and unintentional weight loss.  Lupus symptoms include a butterfly rash and arthralgias.

    Hypothyroidism is more prevalent in women, elderly, and white individuals with causes being classified as either primary (thyroid gland unable to produce thyroid hormone) or secondary (TSH deficiency related to dysfunction of the pituitary or hypothalamus) or tertiary (caused by any disorder that damages the hypothalamus or interferes with hypothalamic-pituitary portal blood flow). Etiologies of primary hypothyroidism include autoimmune thyroiditis, dietary iodine deficiency, thyroidectomy, radioiodine therapy for hyperthyroidism, neck radiation, medications, and infiltrative diseases. Secondary hypothyroidism is much less common, but etiologies include pituitary tumor, postpartum pituitary necrosis (Sheehan syndrome), trauma, hypophysitis, nonpituitary tumors, and inactivating mutations in the gene for either TSH or the TSH receptor.

    In addition to the findings mentioned above, additional patient complaints may include voice changes, hair loss, constipation, muscle cramps, sleep disturbances, menstrual cycle abnormalities, weight gain, depression/anxiety, psychosis, and cognitive dysfunction . Findings on physical exam can include enlarged thyroid gland, slow speech/movements or dull facial expressions, coarse and brittle hair, pallor, and bradycardia.

    Work-up starts a TSH level which is usually elevated and a free T4 level which is usually low. Subclinical cases may have a normal free T4. If suspected, thyroid peroxidase antibodies should be assessed. Additional lab abnormalities which could be present if tested include hyperlipidemia, anemia, and elevations of CK, liver enzymes, BUN, creatinine, or uric acid. Unlike with hyperthyroidism, ultrasound is usually not indicated for hypothyroidism. Asymptomatic screening should be considered in patients with the following risk factors: significant hyperlipidemia, hyponatremia, macrocytic anemia, pericardial or pleural effusions, prior thyroid injury, pituitary or hypothalamic disorders, and a history of autoimmune disorders.

    The primary treatment of hypothyroidism is to give synthetic thyroxine (T4, levothyroxine) with the average adult dosage being 1.6 mcg/kg/day taken ideally 30-60 minutes prior to breakfast. Elderly or patients with coronary artery disease may benefit from starting at a lower dose while obese patients may ultimately require higher doses. A TSH should be redrawn in 4-6 weeks and the dose of T4 increased by 12 to 25 mcg/day every 3-6 weeks as needed until the TSH returns to the normal reference level.

    References:

    1. UpToDate
    2. https://www.ncbi.nlm.nih.gov/pmc/articles/PMC6619426/
    3. https://www.ncbi.nlm.nih.gov/books/NBK519536/
  24. Question 24 of 30
    24. Question

    Which finding differentiates De Quervain’s (subacute) thyroiditis from other types of thyroiditis?

    Correct

    The correct answer is C. Thyroiditis can result in hyperthyroidism or hypothyroidism. Hyperthyroidism is associated with weight loss, heat intolerance and dry skin while cold intolerance is associated with hypothyroidism. Thyroid pain and swelling are also signs of subacute or De Quervain’s thyroiditis.

    Thyroiditis can be divided into painful and painless presentations and further divided by etiologies. Painful categories include subacute, infectious, radiation and palpation or trauma induced. Painless categories include postpartum state, drug-induced, and fibrous.

    Subacute – postviral inflammatory process including Coxsackievirus as well a strong association with the presence of human leukocyte antigen (HLA)-B35.

    Infectious – commonly Staphylococcus or Streptococcus hematogenous spread, usually in an immunocompromised patient or via a fistula. Mycobacterial, fungal, and Pneumocystis infections may be chronic and slower evolving.

    Radiation – usually post radioiodine treatment for Graves’ disease

    Palpation – vigorous manipulation during physical examination, biopsy or neck surgery

    postpartum – within 1 year of spontaneous or induced abortion

    drug-induced – interferon alfa, interleukin-2, amiodarone, or lithium

    fibrous – primary fibrosing disorder called Riedel’s thyroiditis also associated with mediastinal and retroperitoneal fibrosis.

    As mentioned, thyroiditis can result in either hyper or hypo-thyroidism and can start as one and transition to another. History and risk factors are important components of making the diagnosis. Addressing the underlying cause and treating for hyper or hypo-thyroid until normal TSH is achieved. Some may resolve spontaneously.

    References:

    1. https://www.endocrineweb.com/conditions/thyroid/thyroiditis
    2. UpToDate
    Incorrect

    The correct answer is C. Thyroiditis can result in hyperthyroidism or hypothyroidism. Hyperthyroidism is associated with weight loss, heat intolerance and dry skin while cold intolerance is associated with hypothyroidism. Thyroid pain and swelling are also signs of subacute or De Quervain’s thyroiditis.

    Thyroiditis can be divided into painful and painless presentations and further divided by etiologies. Painful categories include subacute, infectious, radiation and palpation or trauma induced. Painless categories include postpartum state, drug-induced, and fibrous.

    Subacute – postviral inflammatory process including Coxsackievirus as well a strong association with the presence of human leukocyte antigen (HLA)-B35.

    Infectious – commonly Staphylococcus or Streptococcus hematogenous spread, usually in an immunocompromised patient or via a fistula. Mycobacterial, fungal, and Pneumocystis infections may be chronic and slower evolving.

    Radiation – usually post radioiodine treatment for Graves’ disease

    Palpation – vigorous manipulation during physical examination, biopsy or neck surgery

    postpartum – within 1 year of spontaneous or induced abortion

    drug-induced – interferon alfa, interleukin-2, amiodarone, or lithium

    fibrous – primary fibrosing disorder called Riedel’s thyroiditis also associated with mediastinal and retroperitoneal fibrosis.

    As mentioned, thyroiditis can result in either hyper or hypo-thyroidism and can start as one and transition to another. History and risk factors are important components of making the diagnosis. Addressing the underlying cause and treating for hyper or hypo-thyroid until normal TSH is achieved. Some may resolve spontaneously.

    References:

    1. https://www.endocrineweb.com/conditions/thyroid/thyroiditis
    2. UpToDate
  25. Question 25 of 30
    25. Question

    Which lab test is most specific to Hashimoto thyroiditis?

    Correct

    The correct answer is A. T3, T4, and TSH are used in the work-up of hyper/hypo-thyroidism. In Hashimoto’s, the most common lab findings include an elevated thyroid-stimulating hormone (TSH), low thyroxine (T4), and increased antithyroid peroxidase (anti-TPO) antibodies.  A patient with evidence of hyperthyroidism and a positive stimulatory TSH receptor antibody (TSI) is diagnosed with Graves’ Disease. Thyroglobulin is a protein produced by both normal and thyroid cancer cells. It is generally used to monitor a patient after surgery for thyroid cancer.

    Hashimoto’s thyroiditis, also called autoimmune or chronic lymphocytic thyroiditis, in which thyroid cells are destroyed. It is the most common type of thyroiditis and the most common cause of hypothyroidism. Hashimoto’s is thought to be a combination of genetic susceptibility and environmental factors with some relationship to Graves’ disease pathophysiologically. It is much more common in females than males. As the thyroid attempts to compensate for inefficiencies, it can become globally enlarged or unilateral enlargement may occur. As the thyroid cells are destroyed, the pituitary compensates by increasing the TSH while the T4 declines because the thyroid can’t make it. The result is hypothyroidism. Many patients will also have anemia and possibly a decrease in renal function, hyponatremia, or hypercholesterolemia.

    Treatment is usually with levothyroxine sodium with a standard dose being 1.6 – 1.8 mcg/kg per day. Lower doses should be used in patients with cardiovascular diseases and the elderly (starting dose 25 mcg/day) with reevaluation in six to eight weeks. Higher doses may be necessary in pregnancy and in patients with short bowel syndrome. Once thyroid hormone replacement is started, the enlargement of the thyroid gland will decline, however, thyroid antibodies may remain detectable for years.

    References:

    1. https://www.thyroid.org/thyroid-function-tests/
    2. https://www.ncbi.nlm.nih.gov/books/NBK459262/
    3. UpToDate
    Incorrect

    The correct answer is A. T3, T4, and TSH are used in the work-up of hyper/hypo-thyroidism. In Hashimoto’s, the most common lab findings include an elevated thyroid-stimulating hormone (TSH), low thyroxine (T4), and increased antithyroid peroxidase (anti-TPO) antibodies.  A patient with evidence of hyperthyroidism and a positive stimulatory TSH receptor antibody (TSI) is diagnosed with Graves’ Disease. Thyroglobulin is a protein produced by both normal and thyroid cancer cells. It is generally used to monitor a patient after surgery for thyroid cancer.

    Hashimoto’s thyroiditis, also called autoimmune or chronic lymphocytic thyroiditis, in which thyroid cells are destroyed. It is the most common type of thyroiditis and the most common cause of hypothyroidism. Hashimoto’s is thought to be a combination of genetic susceptibility and environmental factors with some relationship to Graves’ disease pathophysiologically. It is much more common in females than males. As the thyroid attempts to compensate for inefficiencies, it can become globally enlarged or unilateral enlargement may occur. As the thyroid cells are destroyed, the pituitary compensates by increasing the TSH while the T4 declines because the thyroid can’t make it. The result is hypothyroidism. Many patients will also have anemia and possibly a decrease in renal function, hyponatremia, or hypercholesterolemia.

    Treatment is usually with levothyroxine sodium with a standard dose being 1.6 – 1.8 mcg/kg per day. Lower doses should be used in patients with cardiovascular diseases and the elderly (starting dose 25 mcg/day) with reevaluation in six to eight weeks. Higher doses may be necessary in pregnancy and in patients with short bowel syndrome. Once thyroid hormone replacement is started, the enlargement of the thyroid gland will decline, however, thyroid antibodies may remain detectable for years.

    References:

    1. https://www.thyroid.org/thyroid-function-tests/
    2. https://www.ncbi.nlm.nih.gov/books/NBK459262/
    3. UpToDate
  26. Question 26 of 30
    26. Question

    A patient with hypertension, hypercholesterolemia, and elevated fasting glucose most likely has which disorder?

    Correct

    The correct answer is C. Diabetes insipidus is an endocrine disorder that results in hypernatremia. Klinefelter syndrome is a genetic condition that causes a male to have an extra copy of the X chromosome, resulting in smaller than normal testicles, reduced muscle mass, reduced body and facial hair, and enlarged breast tissue. Minimal change disease is a renal disorder that can lead to nephrotic syndrome resulting in proteinuria, low protein, high cholesterol & triglyceride levels, as well as swelling.

    Metabolic syndrome, also referred to as syndrome X, is a multifactorial disease primarily involving complications secondary to insulin resistance. It is also considered to cause a proinflammatory, prothrombotic state. Its existence increases risk for developing coronary heart disease, stroke, diabetes, fatty liver, and several cancers. Three of the following are required to be present to make the diagnosis:

    • waist circumference ≥ 40″ in men and 35″ in females
    • triglycerides ≥150 mg/dL
    • high-density lipoprotein (HDL) cholesterol <40 mg/dL in males and <50 mg/dL in females
    • Blood pressure ≥130/85 mmHg
    • Fasting glucose ≥100 mg/dL
      • the active treatment for any of the final 4 criteria also counts

    Due to the proinflammatory/prothrombotic state that is also present, metabolic syndrome may also result in elevated levels of C-reactive protein (CRP), interleukin (IL)-6, and plasminogen activator inhibitor. Risk factors for developing the syndrome include obesity, age >60, African American/Mexican American race, postmenopausal, smoking, no alcohol consumption, sedentary lifestyle, and sugar-sweetened beverage consumption.

    Management starts with lifestyle modification including diet and exercise.  Sleep apnea and sleep deprivation should also be considered because they have been associated with the development of metabolic syndrome. Metformin, cholesterol medication and antihypertensives should be started when appropriate.

    References:

    1. UpToDate
    2. https://www.ncbi.nlm.nih.gov/books/NBK459248/
    Incorrect

    The correct answer is C. Diabetes insipidus is an endocrine disorder that results in hypernatremia. Klinefelter syndrome is a genetic condition that causes a male to have an extra copy of the X chromosome, resulting in smaller than normal testicles, reduced muscle mass, reduced body and facial hair, and enlarged breast tissue. Minimal change disease is a renal disorder that can lead to nephrotic syndrome resulting in proteinuria, low protein, high cholesterol & triglyceride levels, as well as swelling.

    Metabolic syndrome, also referred to as syndrome X, is a multifactorial disease primarily involving complications secondary to insulin resistance. It is also considered to cause a proinflammatory, prothrombotic state. Its existence increases risk for developing coronary heart disease, stroke, diabetes, fatty liver, and several cancers. Three of the following are required to be present to make the diagnosis:

    • waist circumference ≥ 40″ in men and 35″ in females
    • triglycerides ≥150 mg/dL
    • high-density lipoprotein (HDL) cholesterol <40 mg/dL in males and <50 mg/dL in females
    • Blood pressure ≥130/85 mmHg
    • Fasting glucose ≥100 mg/dL
      • the active treatment for any of the final 4 criteria also counts

    Due to the proinflammatory/prothrombotic state that is also present, metabolic syndrome may also result in elevated levels of C-reactive protein (CRP), interleukin (IL)-6, and plasminogen activator inhibitor. Risk factors for developing the syndrome include obesity, age >60, African American/Mexican American race, postmenopausal, smoking, no alcohol consumption, sedentary lifestyle, and sugar-sweetened beverage consumption.

    Management starts with lifestyle modification including diet and exercise.  Sleep apnea and sleep deprivation should also be considered because they have been associated with the development of metabolic syndrome. Metformin, cholesterol medication and antihypertensives should be started when appropriate.

    References:

    1. UpToDate
    2. https://www.ncbi.nlm.nih.gov/books/NBK459248/
  27. Question 27 of 30
    27. Question

    An incidental thyroid nodule is found on physical exam. What is the next most appropriate step in the workup?

    Correct

    The correct answer is E. Fine needle aspiration (FNA) is indicated if the TSH is normal or elevated or if the nodule is nonfunctional. A free T4 (FT4), a T3, and a radionuclide scan are ordered if the TSH is below normal range. A Free T3 is generally not utilized. The initial TSH should be accompanied by a thyroid ultrasound which will help in determining if it meets criteria for fine needle aspiration.

    If the TSH is below normal, a radionuclide scan, a free T4, and T3 should be ordered. Oral or IV administration of radioactive iodine will accumulate in the thyroid gland and can be visualized using nuclear imaging. If a nodule is producing too much hormone, it appears as a “hot spot” on imaging making it a “functional” nodule. A non-functional or hypo-functional nodule will appear as a “cold spot”. Cold (non-functional) nodules are more likely to be cancerous because cancer cells are immature and don’t accumulate the iodine as well as normal thyroid tissue. Cold spots can also be caused by cysts, which an ultrasound can help differentiate. If nonfunctional, and sonographic criteria is met, FNA is indicated. FNA is also indicated if the initial TSH is normal or elevated. A functional nodule with a normal FT4 and T3 indicates subclinical hyperthyroidism and should be observed/monitored. If either is elevated, treat for hyperthyroidism.

    A solitary nodule is a higher risk for malignancy than a multinodular thyroid. Benign nodules can be monitored with ultrasound every 12 to 24 months. Malignant nodules (papillary, medullary, lymphoma, anaplastic, and metastatic cancers) require surgery.

    References:

    1. UpToDate
    2. https://www.ncbi.nlm.nih.gov/books/NBK535422/
    3. https://www.medicinenet.com/thyroid_nodules/article.htm
    Incorrect

    The correct answer is E. Fine needle aspiration (FNA) is indicated if the TSH is normal or elevated or if the nodule is nonfunctional. A free T4 (FT4), a T3, and a radionuclide scan are ordered if the TSH is below normal range. A Free T3 is generally not utilized. The initial TSH should be accompanied by a thyroid ultrasound which will help in determining if it meets criteria for fine needle aspiration.

    If the TSH is below normal, a radionuclide scan, a free T4, and T3 should be ordered. Oral or IV administration of radioactive iodine will accumulate in the thyroid gland and can be visualized using nuclear imaging. If a nodule is producing too much hormone, it appears as a “hot spot” on imaging making it a “functional” nodule. A non-functional or hypo-functional nodule will appear as a “cold spot”. Cold (non-functional) nodules are more likely to be cancerous because cancer cells are immature and don’t accumulate the iodine as well as normal thyroid tissue. Cold spots can also be caused by cysts, which an ultrasound can help differentiate. If nonfunctional, and sonographic criteria is met, FNA is indicated. FNA is also indicated if the initial TSH is normal or elevated. A functional nodule with a normal FT4 and T3 indicates subclinical hyperthyroidism and should be observed/monitored. If either is elevated, treat for hyperthyroidism.

    A solitary nodule is a higher risk for malignancy than a multinodular thyroid. Benign nodules can be monitored with ultrasound every 12 to 24 months. Malignant nodules (papillary, medullary, lymphoma, anaplastic, and metastatic cancers) require surgery.

    References:

    1. UpToDate
    2. https://www.ncbi.nlm.nih.gov/books/NBK535422/
    3. https://www.medicinenet.com/thyroid_nodules/article.htm
  28. Question 28 of 30
    28. Question

    How many weeks into the gestational period does insulin resistance significantly increase?

    Correct

    The correct answer is 24 weeks. During pregnancy the placenta secretes growth and corticotropin-releasing hormones, placental lactogen, prolactin, and progesterone. All of which are intended to ensure an ample supply of nutrients to the fetus but contribute to an insulin-resistant state known as gestational diabetes (GDM) Risk factors for developing GDM include prior pregnancy with GDM, family hx of DM, BMI >30, age > 40, and prior birth weight of an infant ≥ 9 pounds.

    Complications of gestational diabetes include preeclampsia and large newborns possibly requiring cesarean birth. After the pregnancy has concluded, these patients are at increased risk of developing type 2 diabetes. Children of mothers with gestational diabetes are at increased risk of developing obesity, abnormal glucose tolerance, hypertension, and metabolic syndrome.

    A hemoglobin A1C at the initial prenatal visit can be considered, especially for those with any of the risk factors discussed, otherwise, initial testing should start at 24-28 weeks with a 2 step process. First, 50 grams of oral glucose is given, and a blood sugar is assessed in 1 hour. If positive a fasting glucose is checked followed by a 100 gram oral glucose solution with blood sugar measurements one, two, and three hours after administration. A positive test is when an elevated glucose is detected at two or more time points. An alternative, 1 step 75 gram test is also utilized.

    Once diagnosed, lifestyle changes and insulin are the mainstays of management. Blood sugar recommendations are as follows:

    • Fasting and preprandial <95 mg/dL
    • 1 hour postprandial <140 mg/dL
    • 2 hour postprandial <120 mg/dL

    Typical insulin dosing is 0.7 to 2 units per kg with titration based upon frequent self-monitoring. At least four daily glucose measurements are recommended including both fasting and either one or two hours postprandial being used. After delivery, a glucose tolerance test is recommended at 4–12 weeks. Women should also be tested every 1–3 years thereafter if the tolerance test was normal.

     

    References:

    1. UpToDate
    2. https://care.diabetesjournals.org/content/44/Supplement_1/S200
    Incorrect

    The correct answer is 24 weeks. During pregnancy the placenta secretes growth and corticotropin-releasing hormones, placental lactogen, prolactin, and progesterone. All of which are intended to ensure an ample supply of nutrients to the fetus but contribute to an insulin-resistant state known as gestational diabetes (GDM) Risk factors for developing GDM include prior pregnancy with GDM, family hx of DM, BMI >30, age > 40, and prior birth weight of an infant ≥ 9 pounds.

    Complications of gestational diabetes include preeclampsia and large newborns possibly requiring cesarean birth. After the pregnancy has concluded, these patients are at increased risk of developing type 2 diabetes. Children of mothers with gestational diabetes are at increased risk of developing obesity, abnormal glucose tolerance, hypertension, and metabolic syndrome.

    A hemoglobin A1C at the initial prenatal visit can be considered, especially for those with any of the risk factors discussed, otherwise, initial testing should start at 24-28 weeks with a 2 step process. First, 50 grams of oral glucose is given, and a blood sugar is assessed in 1 hour. If positive a fasting glucose is checked followed by a 100 gram oral glucose solution with blood sugar measurements one, two, and three hours after administration. A positive test is when an elevated glucose is detected at two or more time points. An alternative, 1 step 75 gram test is also utilized.

    Once diagnosed, lifestyle changes and insulin are the mainstays of management. Blood sugar recommendations are as follows:

    • Fasting and preprandial <95 mg/dL
    • 1 hour postprandial <140 mg/dL
    • 2 hour postprandial <120 mg/dL

    Typical insulin dosing is 0.7 to 2 units per kg with titration based upon frequent self-monitoring. At least four daily glucose measurements are recommended including both fasting and either one or two hours postprandial being used. After delivery, a glucose tolerance test is recommended at 4–12 weeks. Women should also be tested every 1–3 years thereafter if the tolerance test was normal.

     

    References:

    1. UpToDate
    2. https://care.diabetesjournals.org/content/44/Supplement_1/S200
  29. Question 29 of 30
    29. Question

    A 43 year old woman presents with a history of poorly controlled hypertension despite trial of three blood pressure medications. Recent imaging reveals an adenoma on her right adrenal gland. The patient refuses surgical intervention. Due to the untreated adenoma, you should further counsel the patient on the development of headaches, fatigue, muscle weakness and what electrolyte abnormality?

    Correct

    Answer: B

    Secondary hypertension can have numerous causes. The cause of this patient’s hypertension is primary hyperaldosteronism (Conn’s disease) from a benign aldosterone secreting adenoma in the adrenal cortex. A smaller subset of the patients with primary hyperaldosteronism have nodular hyperplasia.1 This diagnosis is often made around 40-50 years old.2 Most of the time primary hyperaldosteronism is caused by adrenal adenoma or nodular hyperplasia but in rarer cases it may be due to autosomal dominant familial hyperaldosteronism and adrenal carcinomas.2

    The aldosterone-secreting adenoma leads to an increased aldosterone effect. The most notable electrolyte abnormality seen is hypokalemia because aldosterone acts on the cortical collecting duct and will prompt the overexcretion of potassium into tubular fluid while retaining sodium and water. This sodium and water retention leads to increased blood pressure.3

    The history of at least three different medications for blood pressure control without adequate suppression is a common presentation in the setting of hyperaldosteronism. The patient may experience symptoms related to their hypertension such as headaches, dizziness, or blurry vision.3 There may be physical exam findings due to hypertensive complications such as carotid bruits, hypertensive encephalopathy, hypertensive retinal changes, and cardiac failure.4 They may also experience symptoms related to hypokalemia such as numbness, fatigue, muscle cramps, or increased thirst and urination.3 These patients may develop hypokalemia when treated with potassium-wasting diuretics.4 While history is often the most helpful to diagnosis, the physical exam findings may include weakness, abdominal distention, or even ileus from hypokalemia.

    In working this up, serum potassium, bicarbonate, sodium and magnesium levels should be obtained. Along with this, the plasma aldosterone/plasma renin activity ratio is measured. To confirm the abnormality, a serum aldosterone level and 24-hour urine collection are performed. The imaging used initially is a thin-sliced adrenal CT without contrast. If the subtype of primary hyperaldosteronism is in question, a Lasix simulation test, postural stimulation test, or diurnal rhythm of aldosterone level is performed.4

    Unilateral adrenalectomy of the gland which contains the adenoma will likely cure the hypertension. The optimal medical management has been achieved with spironolactone and a thiazide diuretic. This is due to the mineralocorticoid antagonism that spironolactone works by. If a patient is experiencing the unwanted side effects of spironolactone, such as gynecomastia, eplerenone can be used due to its specificity for the aldosterone receptor without the antiandrogen effects.4

    As mentioned, the electrolyte abnormality seen in primary hyperaldosteronism is hypokalemia. You would likely not see hyperkalemia because the pathology leads to potassium wasting at the level of the cortical collecting duct due to increased aldosterone. The patient would also not likely experience hyponatremia and patients are retaining water and sodium. Hypercalcemia is not seen in this condition, and there have been case reports of hypocalcemia in the setting of primary hyperaldosteronism.5

    References

    1. McCance KL, Huether SE. Pathophysiology: the Biologic Basis for Disease in Adults and Children. 7th ed. St. Louis: Mosby; 2015.

     

    1. Primary aldosteronism. DynaMed Plus [Internet]. November 2018. https://www.dynamed.com/topics/dmp~AN~T114523. Accessed January 25, 2018.

     

    1. Primary Hyperaldosteronism (Conn’s Syndrome). Adrenal Center. http://columbiasurgery.org/conditions-and-treatments/primary-hyperaldosteronism-conns-syndrome. Accessed February 2, 2019.

     

    1. Uwaifo GI. Primary Aldosteronism Treatment & Management. Sickle Cell Anemia Differential Diagnoses. https://emedicine.medscape.com/article/127080-treatment. Published March 2, 2018. Accessed January 27, 2019.

     

    1. Pai SG, Shivashankara K, Pandit V, Sheshadri S. Symptomatic Hypocalcemia in Primary Hyperaldosteronism: A Case Report. Journal of Korean Medical Science. 2009;24(6):1220-1223. doi:10.3346/jkms.2009.24.6.1220.
    Incorrect

    Answer: B

    Secondary hypertension can have numerous causes. The cause of this patient’s hypertension is primary hyperaldosteronism (Conn’s disease) from a benign aldosterone secreting adenoma in the adrenal cortex. A smaller subset of the patients with primary hyperaldosteronism have nodular hyperplasia.1 This diagnosis is often made around 40-50 years old.2 Most of the time primary hyperaldosteronism is caused by adrenal adenoma or nodular hyperplasia but in rarer cases it may be due to autosomal dominant familial hyperaldosteronism and adrenal carcinomas.2

    The aldosterone-secreting adenoma leads to an increased aldosterone effect. The most notable electrolyte abnormality seen is hypokalemia because aldosterone acts on the cortical collecting duct and will prompt the overexcretion of potassium into tubular fluid while retaining sodium and water. This sodium and water retention leads to increased blood pressure.3

    The history of at least three different medications for blood pressure control without adequate suppression is a common presentation in the setting of hyperaldosteronism. The patient may experience symptoms related to their hypertension such as headaches, dizziness, or blurry vision.3 There may be physical exam findings due to hypertensive complications such as carotid bruits, hypertensive encephalopathy, hypertensive retinal changes, and cardiac failure.4 They may also experience symptoms related to hypokalemia such as numbness, fatigue, muscle cramps, or increased thirst and urination.3 These patients may develop hypokalemia when treated with potassium-wasting diuretics.4 While history is often the most helpful to diagnosis, the physical exam findings may include weakness, abdominal distention, or even ileus from hypokalemia.

    In working this up, serum potassium, bicarbonate, sodium and magnesium levels should be obtained. Along with this, the plasma aldosterone/plasma renin activity ratio is measured. To confirm the abnormality, a serum aldosterone level and 24-hour urine collection are performed. The imaging used initially is a thin-sliced adrenal CT without contrast. If the subtype of primary hyperaldosteronism is in question, a Lasix simulation test, postural stimulation test, or diurnal rhythm of aldosterone level is performed.4

    Unilateral adrenalectomy of the gland which contains the adenoma will likely cure the hypertension. The optimal medical management has been achieved with spironolactone and a thiazide diuretic. This is due to the mineralocorticoid antagonism that spironolactone works by. If a patient is experiencing the unwanted side effects of spironolactone, such as gynecomastia, eplerenone can be used due to its specificity for the aldosterone receptor without the antiandrogen effects.4

    As mentioned, the electrolyte abnormality seen in primary hyperaldosteronism is hypokalemia. You would likely not see hyperkalemia because the pathology leads to potassium wasting at the level of the cortical collecting duct due to increased aldosterone. The patient would also not likely experience hyponatremia and patients are retaining water and sodium. Hypercalcemia is not seen in this condition, and there have been case reports of hypocalcemia in the setting of primary hyperaldosteronism.5

    References

    1. McCance KL, Huether SE. Pathophysiology: the Biologic Basis for Disease in Adults and Children. 7th ed. St. Louis: Mosby; 2015.

     

    1. Primary aldosteronism. DynaMed Plus [Internet]. November 2018. https://www.dynamed.com/topics/dmp~AN~T114523. Accessed January 25, 2018.

     

    1. Primary Hyperaldosteronism (Conn’s Syndrome). Adrenal Center. http://columbiasurgery.org/conditions-and-treatments/primary-hyperaldosteronism-conns-syndrome. Accessed February 2, 2019.

     

    1. Uwaifo GI. Primary Aldosteronism Treatment & Management. Sickle Cell Anemia Differential Diagnoses. https://emedicine.medscape.com/article/127080-treatment. Published March 2, 2018. Accessed January 27, 2019.

     

    1. Pai SG, Shivashankara K, Pandit V, Sheshadri S. Symptomatic Hypocalcemia in Primary Hyperaldosteronism: A Case Report. Journal of Korean Medical Science. 2009;24(6):1220-1223. doi:10.3346/jkms.2009.24.6.1220.
  30. Question 30 of 30
    30. Question

    A 19-year-old African American woman with no significant past medical history presents to your primary care office with complaints of her “heart racing” and sweating/feeling very hot despite her thermostat being set at 70F. She has noticed this consistently for the past few months and reports it seems to be getting worse, she now wears only short sleeve shirts to work. She proudly reports having lost 15 pounds and states she has “barely had to try”. On exam, you notice mild exophthalmoses and note an enlarged, smooth thyroid during palpation. She denies any dysphagia, hoarseness, excessive caffeine intake, new medications. You suspect Graves’ Disease. Which of the following laboratory results would confirm the diagnosis?

    Correct

    ANSWER: D

    Explanation: This clinical vignette is pointing towards a diagnosis of Graves’ disease. The symptoms of heat intolerance, tachycardia, and unintentional weight loss raise your clinical suspicion for hyperthyroidism. Physical exam findings of exophthalmoses and a large thyroid further support the diagnosis. The next step in confirming a diagnosis of Graves’ disease would be blood work with a thyroid panel including TSH, free T3/T4. In Graves’ disease, the thyroid stimulating hormone which comes from the pituitary gland is not elevated. It is, in fact, low due to an autoimmune process which creates autoantibodies against TSH receptors. Elevations of circulating free T3 and T4 come directly from the overactivity of the thyroid gland itself.  Elevated thyroid stimulating hormone, high free T4, high free T3 (Option A) may occur in the case of a TSH producing pituitary adenoma. Elevated thyroid stimulating hormone, low free T4, low free T3 (Option B) may occur in the case of primary hypothyroidism. Low thyroid stimulating hormone, high free T4, normal free T3 (Option C) may occur in the case of thyroiditis, inflammation of the thyroid gland.

    DISCUSSION OF GRAVES’ DISEASE

     

    Graves’ disease is the most common cause of hyperthyroidism in the United States, affecting roughly one in every 200 people nationwide. Most commonly, Graves’ disease begins in adolescence, usually after age 11, and it is five times more common in females than in males (Williams). It is seen most often in non-Hispanic blacks. Currently, Graves’ disease is on the rise, with case reports in Denmark doubling between the years 1988 and 2012 (uptodate). It has the potential to be socially embarrassing, physically painful, and, at its most extreme, life threatening. However, Graves’ disease and its complications can be managed easily and completely through medical and surgical measures. Here, we will explore the pathogenesis, etiology, symptoms, signs, complications, diagnosis, and treatment options for Graves’ disease.

    In a healthy, normally functioning thyroid gland, thyrotropin (TSH) is released from the pituitary gland and stimulates the thyroid gland to produce of triiodothyronine (T3) and thyroxine (T4) from dietary iodine (uptodate). The thyroid gland then releases T3 and T4 which have important effects on neural and somatic development in infants and work within almost every organ system of adults. Specifically, T3 and T4 play a major role in bone development in children, maintenance of bone density throughout life, controlling heart rate, determining basal metabolic rate and managing body weight.

    In Graves’ disease, this process is derailed. According to Uptodate, it is thought that when a patient develops Graves’ disease it is because there was an insult to the thyroid gland which stimulated an overactive autoimmune response. Some insults that have been identified as precipitating events are infection (congenital rubella and Hepatitis C), psychologic stress, pregnancy, drugs that contain iodine such as amiodarone, and smoking. It is also thought that genetic susceptibility plays a role because Graves’ disease clusters in families. Additionally, there are multiple alleles that are commonly found in patients with Graves’: G allele of CTLA-4 gene and certain alleles on chromosome 6 of HLA (uptodate). Female gender is yet another predisposing factor.

    When a person has the right combination of predisposing factors and experiences a precipitating event, they will begin to produce an autoantibody to the thyrotropin receptor (TRAb). This TRAb turns the sodium-iodide symporter within the thyroid on, causing increased uptake of iodide by thyroid tissue, thus stimulating thyroid gland growth and increased synthesis and secretion of T3/T4 in the absence of TSH from the pituitary gland (uptodate). TRAb also plays a role in the secretion of long-acting thyroid stimulator immunoglobulin which is found in abnormally high concentrations in patients with Graves’ disease’ and acts to stimulate the thyroid. Overstimulation leads to follicular hyperplasia and diffuse enlargement of the thyroid gland. Lymphocytic infiltration (B and T cells invading the thyroid) also plays a role in thyroid overgrowth (uptodate).

    Patients with Graves’ disease often experience a constellation of mild signs and symptoms. Often, the disease goes unnoticed for quite some time and may be incidentally found by a primary care provider after routine questioning. The classic patient presentation includes reported symptoms of tachycardia, heat intolerance, failure to gain weight or unintentional weight loss, loose stools, dysphagia, tremulousness, hyperactivity, grittiness in eyes, and thinning of hair. (Babcock O’Connell). Adolescents may experience a height increase that is greater than expected. Girls often experience oligomenorrhea or amenorrhea. Depending on how advanced the disease is, a patient may develop a visible growth in the size of their thyroid gland, termed a “goiter”. This goiter may look like a softball, or even larger, within the neck. It can be embarrassing to patients and uncomfortable. It is also concerning as patients may experience dysphagia to both solids and liquids and dyspnea due to pressure on their airway (Williams).

    A healthcare provider may note a bruit over the thyroid, lid lag, a “stare” (widely opened eyes), and exophthalmoses. A finding that is exclusively found in Graves’ hyperthyroidism is pretibial myxedema (Engorn). This is a nonpitting erythematous discoloration with pink/brown plaques on a patient’s shins. Patients will exhibit hyperreflexia when examiners check deep tendon reflexes (Williams).

    Patients with Graves’ disease have increased cardiac output due to overexcretion of T3 and T4 which causes an increased cardiac contractility and increased peripheral oxygen demands (uptodate). They experience tachycardia, a wide pulse pressure, and decreased peripheral vascular resistance. They may have systolic hypertension, atrial fibrillation or mitral valve prolapse later in adulthood (Babcock O’Connell). If Graves’ disease goes uncontrolled for long enough, the extra demand on the heart can cause cardiomyopathy and a high-output heart failure. Patients may experience classic symptoms of heart failure such as increased jugular venous distention, edema, fatigue, coughing, wheezing, dyspnea on exertion, and exercise intolerance.

    If, after a thorough history and physical exam, the provider’s suspicion for Graves’ disease is high, confirmation of the diagnosis is quite simple and can be done by a blood draw. Initial laboratory studies will confirm a hyperthyroid state by revealing low thyrotropin (TSH) and high levels of free triiodothyronine (fT3) and thyroxine (fT4). Additional work up will confirm Graves’ as the etiology if thyroid-stimulating immunoglobulin (TSI), thyroid peroxidase, and thyroglobulin antibody are confirmed via a functional assay. Additionally, providers can confirm the diagnosis through a radionucleotide uptake and scan. In Graves’, the thyroid will show a diffuse uptake throughout the thyroid gland (uptodate).

    As a complete workup is carried out, a provider will likely order a CBC and may note a normochromic normocytic anemia. This is common in hyperthyroidism because the red blood cell mass is increased, but the overall plasma volume is increased more (uptodate). Another common finding with additional workup is osteopenia. Thyroid hormone increases bone resorption, so having high free T3 and T4 causes abnormally high bone resorption and weakened bones (uptodate). Thymic enlargement may also be noted. It is unknown why the thymus becomes enlarged, but it is commonly found in patients with Graves’ hyperthyroidism.

    Treatment for Graves’ disease can vary depending on the patient and the extent of their disease. Often, medical management is appropriate, however some patients may choose to undergo radioactive iodine ablation or surgical removal of their thyroid.

    • The first line treatment is antithyroid drug therapy with methimazole or propylthiouracil. These medications have potential to make a patient euthyroid for the rest of their life. There is data that suggests patients may need to take these medications for up to ten years to achieve full remission. However, other data suggests that with only two years of treatment, some patients become euthyroid for life (uptodate).
    • Radioactive iodine (RAI) will destroy the thyroid and completely cure patients of hyperthyroidism, but it will make them hypothyroid. They will require lifelong treatment with levothyroxine (Williams).
    • Surgical removal of the thyroid will also cure hyperthyroidism and does not expose the patient to radiation. There are surgical risks and the patient will be hypothyroid for life, requiring levothyroxine supplementation. It is usually used after a patient has failed medical management. Surgical removal of the thyroid is preferred if the goiter is very large (Engorn). Otherwise, deciding between RAI and surgical intervention after failing medical management is often largely based on patient preference.

    A patient may also choose to begin treatment with medical management with a plan to move forward with more definitive treatment through RAI or surgical intervention in the near future. Additionally, most adult patients require a beta blocker (usually propranolol) to relieve symptoms of tachycardia, palpitations, anxiety, tremor, and heat intolerance (Engorn).

    Regardless of treatment method, all patients with Graves’ disease should have lifelong monitoring of thyroid function. Patients have risk of becoming hyperthyroid due to any of the precipitating events discussed above (infection, stress, pregnancy, etc.) while also being at the risk of becoming hypothyroid due to incorrectly dosed medications or a change in their baseline thyroid function. For these reasons, they should be monitored through a primary care provider or endocrinologist regularly.

     

    REFERENCES

     

    Babcock O’Connell, C. and Cogan-Drew, T. A Comprehensive Review for the Certification and Recertification Examinations for Physician Assistants. 6th edition. Wolters Kluwer. 2018. Pg 254-256

     

    Engorn, B. and Flerlage, J. The Harriet Lane Handbook. 20th Edition. Elsevier. 2015. Pg 223.

     

    Uptodate.com. (2019). UpToDate. [online] [Accessed 12-17 September, 2019]

     

    Williams, D. PANCE PREP PEARLS: A Medical Study and Review Guide for the PANCE, PANRE, and Medical Examinations. 2nd edition. CreateSpace Independent Publisher. 2017. Pg 308-312.

     

    Incorrect

    ANSWER: D

    Explanation: This clinical vignette is pointing towards a diagnosis of Graves’ disease. The symptoms of heat intolerance, tachycardia, and unintentional weight loss raise your clinical suspicion for hyperthyroidism. Physical exam findings of exophthalmoses and a large thyroid further support the diagnosis. The next step in confirming a diagnosis of Graves’ disease would be blood work with a thyroid panel including TSH, free T3/T4. In Graves’ disease, the thyroid stimulating hormone which comes from the pituitary gland is not elevated. It is, in fact, low due to an autoimmune process which creates autoantibodies against TSH receptors. Elevations of circulating free T3 and T4 come directly from the overactivity of the thyroid gland itself.  Elevated thyroid stimulating hormone, high free T4, high free T3 (Option A) may occur in the case of a TSH producing pituitary adenoma. Elevated thyroid stimulating hormone, low free T4, low free T3 (Option B) may occur in the case of primary hypothyroidism. Low thyroid stimulating hormone, high free T4, normal free T3 (Option C) may occur in the case of thyroiditis, inflammation of the thyroid gland.

    DISCUSSION OF GRAVES’ DISEASE

     

    Graves’ disease is the most common cause of hyperthyroidism in the United States, affecting roughly one in every 200 people nationwide. Most commonly, Graves’ disease begins in adolescence, usually after age 11, and it is five times more common in females than in males (Williams). It is seen most often in non-Hispanic blacks. Currently, Graves’ disease is on the rise, with case reports in Denmark doubling between the years 1988 and 2012 (uptodate). It has the potential to be socially embarrassing, physically painful, and, at its most extreme, life threatening. However, Graves’ disease and its complications can be managed easily and completely through medical and surgical measures. Here, we will explore the pathogenesis, etiology, symptoms, signs, complications, diagnosis, and treatment options for Graves’ disease.

    In a healthy, normally functioning thyroid gland, thyrotropin (TSH) is released from the pituitary gland and stimulates the thyroid gland to produce of triiodothyronine (T3) and thyroxine (T4) from dietary iodine (uptodate). The thyroid gland then releases T3 and T4 which have important effects on neural and somatic development in infants and work within almost every organ system of adults. Specifically, T3 and T4 play a major role in bone development in children, maintenance of bone density throughout life, controlling heart rate, determining basal metabolic rate and managing body weight.

    In Graves’ disease, this process is derailed. According to Uptodate, it is thought that when a patient develops Graves’ disease it is because there was an insult to the thyroid gland which stimulated an overactive autoimmune response. Some insults that have been identified as precipitating events are infection (congenital rubella and Hepatitis C), psychologic stress, pregnancy, drugs that contain iodine such as amiodarone, and smoking. It is also thought that genetic susceptibility plays a role because Graves’ disease clusters in families. Additionally, there are multiple alleles that are commonly found in patients with Graves’: G allele of CTLA-4 gene and certain alleles on chromosome 6 of HLA (uptodate). Female gender is yet another predisposing factor.

    When a person has the right combination of predisposing factors and experiences a precipitating event, they will begin to produce an autoantibody to the thyrotropin receptor (TRAb). This TRAb turns the sodium-iodide symporter within the thyroid on, causing increased uptake of iodide by thyroid tissue, thus stimulating thyroid gland growth and increased synthesis and secretion of T3/T4 in the absence of TSH from the pituitary gland (uptodate). TRAb also plays a role in the secretion of long-acting thyroid stimulator immunoglobulin which is found in abnormally high concentrations in patients with Graves’ disease’ and acts to stimulate the thyroid. Overstimulation leads to follicular hyperplasia and diffuse enlargement of the thyroid gland. Lymphocytic infiltration (B and T cells invading the thyroid) also plays a role in thyroid overgrowth (uptodate).

    Patients with Graves’ disease often experience a constellation of mild signs and symptoms. Often, the disease goes unnoticed for quite some time and may be incidentally found by a primary care provider after routine questioning. The classic patient presentation includes reported symptoms of tachycardia, heat intolerance, failure to gain weight or unintentional weight loss, loose stools, dysphagia, tremulousness, hyperactivity, grittiness in eyes, and thinning of hair. (Babcock O’Connell). Adolescents may experience a height increase that is greater than expected. Girls often experience oligomenorrhea or amenorrhea. Depending on how advanced the disease is, a patient may develop a visible growth in the size of their thyroid gland, termed a “goiter”. This goiter may look like a softball, or even larger, within the neck. It can be embarrassing to patients and uncomfortable. It is also concerning as patients may experience dysphagia to both solids and liquids and dyspnea due to pressure on their airway (Williams).

    A healthcare provider may note a bruit over the thyroid, lid lag, a “stare” (widely opened eyes), and exophthalmoses. A finding that is exclusively found in Graves’ hyperthyroidism is pretibial myxedema (Engorn). This is a nonpitting erythematous discoloration with pink/brown plaques on a patient’s shins. Patients will exhibit hyperreflexia when examiners check deep tendon reflexes (Williams).

    Patients with Graves’ disease have increased cardiac output due to overexcretion of T3 and T4 which causes an increased cardiac contractility and increased peripheral oxygen demands (uptodate). They experience tachycardia, a wide pulse pressure, and decreased peripheral vascular resistance. They may have systolic hypertension, atrial fibrillation or mitral valve prolapse later in adulthood (Babcock O’Connell). If Graves’ disease goes uncontrolled for long enough, the extra demand on the heart can cause cardiomyopathy and a high-output heart failure. Patients may experience classic symptoms of heart failure such as increased jugular venous distention, edema, fatigue, coughing, wheezing, dyspnea on exertion, and exercise intolerance.

    If, after a thorough history and physical exam, the provider’s suspicion for Graves’ disease is high, confirmation of the diagnosis is quite simple and can be done by a blood draw. Initial laboratory studies will confirm a hyperthyroid state by revealing low thyrotropin (TSH) and high levels of free triiodothyronine (fT3) and thyroxine (fT4). Additional work up will confirm Graves’ as the etiology if thyroid-stimulating immunoglobulin (TSI), thyroid peroxidase, and thyroglobulin antibody are confirmed via a functional assay. Additionally, providers can confirm the diagnosis through a radionucleotide uptake and scan. In Graves’, the thyroid will show a diffuse uptake throughout the thyroid gland (uptodate).

    As a complete workup is carried out, a provider will likely order a CBC and may note a normochromic normocytic anemia. This is common in hyperthyroidism because the red blood cell mass is increased, but the overall plasma volume is increased more (uptodate). Another common finding with additional workup is osteopenia. Thyroid hormone increases bone resorption, so having high free T3 and T4 causes abnormally high bone resorption and weakened bones (uptodate). Thymic enlargement may also be noted. It is unknown why the thymus becomes enlarged, but it is commonly found in patients with Graves’ hyperthyroidism.

    Treatment for Graves’ disease can vary depending on the patient and the extent of their disease. Often, medical management is appropriate, however some patients may choose to undergo radioactive iodine ablation or surgical removal of their thyroid.

    • The first line treatment is antithyroid drug therapy with methimazole or propylthiouracil. These medications have potential to make a patient euthyroid for the rest of their life. There is data that suggests patients may need to take these medications for up to ten years to achieve full remission. However, other data suggests that with only two years of treatment, some patients become euthyroid for life (uptodate).
    • Radioactive iodine (RAI) will destroy the thyroid and completely cure patients of hyperthyroidism, but it will make them hypothyroid. They will require lifelong treatment with levothyroxine (Williams).
    • Surgical removal of the thyroid will also cure hyperthyroidism and does not expose the patient to radiation. There are surgical risks and the patient will be hypothyroid for life, requiring levothyroxine supplementation. It is usually used after a patient has failed medical management. Surgical removal of the thyroid is preferred if the goiter is very large (Engorn). Otherwise, deciding between RAI and surgical intervention after failing medical management is often largely based on patient preference.

    A patient may also choose to begin treatment with medical management with a plan to move forward with more definitive treatment through RAI or surgical intervention in the near future. Additionally, most adult patients require a beta blocker (usually propranolol) to relieve symptoms of tachycardia, palpitations, anxiety, tremor, and heat intolerance (Engorn).

    Regardless of treatment method, all patients with Graves’ disease should have lifelong monitoring of thyroid function. Patients have risk of becoming hyperthyroid due to any of the precipitating events discussed above (infection, stress, pregnancy, etc.) while also being at the risk of becoming hypothyroid due to incorrectly dosed medications or a change in their baseline thyroid function. For these reasons, they should be monitored through a primary care provider or endocrinologist regularly.

     

    REFERENCES

     

    Babcock O’Connell, C. and Cogan-Drew, T. A Comprehensive Review for the Certification and Recertification Examinations for Physician Assistants. 6th edition. Wolters Kluwer. 2018. Pg 254-256

     

    Engorn, B. and Flerlage, J. The Harriet Lane Handbook. 20th Edition. Elsevier. 2015. Pg 223.

     

    Uptodate.com. (2019). UpToDate. [online] [Accessed 12-17 September, 2019]

     

    Williams, D. PANCE PREP PEARLS: A Medical Study and Review Guide for the PANCE, PANRE, and Medical Examinations. 2nd edition. CreateSpace Independent Publisher. 2017. Pg 308-312.

     

Proudly powered by WordPress | Theme: Motif by WordPress.com.